Sample Test Questions: Krok 2

You might also like

Download as pdf or txt
Download as pdf or txt
You are on page 1of 29

Sample test questions

Krok 2
Medicine
()
Терапевтичний профiль 2

1. A 25-year-old woman has been A. Transfer into the inpatient narcology


suffering from diabetes mellitus since she department
was 9. She was admitted into the nephrology B. Continue the treatment in the therapeutic
unit with significant edemas of the face, arms, department
and legs. Blood pressure - 200/110 mm Hg, C. Transfer into the neuroresuscitation
Hb- 90 g/L, blood creatinine - 850 mcmol/L, department
urine proteins - 1.0 g/L, leukocytes - 10-15 in D. Compulsory medical treatment for
the vision field. Glomerular filtration rate - alcoholism
10 mL/min. What tactics should the doctor E. Discharge from the hospital
choose?
5. After eating shrimps, a 25-year-old man
A. Transfer into the hemodialysis unit suddenly developed skin itching, some areas
B. Active conservative therapy for diabetic of his skin became hyperemic or erupted into
nephropathy vesicles. Make the diagnosis:
C. Dietotherapy
D. Transfer into the endocrinology clinic A. Acute urticaria
E. Renal transplantation B. Hemorrhagic vasculitis (Henoch-Schonlein
purpura)
2. A 59-year-old woman was brought into the C. Urticaria pigmentosa
rheumatology unit. Extremely severe case D. Psoriasis
of scleroderma is suspected. Objectively she E. Scabies
presents with malnourishment, ”mask-like”
face, and acro-osteolysis. Blood: erythrocytes 6. A 25-year-old woman complains of fatigue,
- 2.2 · 109 /L, erythrocyte sedimentation rate - dizziness, hemorrhagic rashes on the skin.
40 mm/hour. Urine: elevated levels of free She has been presenting with these signs for a
oxyproline. Name one of the most likely month. Blood test: erythrocytes - 1.0 · 1012 /L,
pathogenetic links in this case: Hb- 37 g/L, color index - 1.1, leukocytes -
1.2 · 109 /L, platelets - 42 · 109 /L. What analysis
A. Formation of antibodies to collagen would be the most advisable for diagnosis-
B. Formation of antibodies to native DNA making in this case?
C. Formation of antibodies to blood
corpuscles A. Sternal puncture (bone marrow biopsy)
D. Formation of antibodies to transversely B. Splenic biopsy
striated muscles C. Liver biopsy
E. Formation of antibodies to vessel wall D. Coagulation studies
E. US of the gastrointestinal tract
3. A 34-year-old man on the 3rd day of
ceftriaxone treatment for acute otitis (daily 7. A 35-year-old man complains of rapidly
dosage - 2 grams) developed diarrhea incresing fatigue, palpitations, ”visual snow”,
occurring 5-6 times per day. Feces are without and dizziness. He has a history of peptic
mucus or blood admixtures. Temperature is ulcer of the stomach. Objectively the skin
36.6o C . Gregersen reaction (occult blood in is pale. Vesicular respiration is observed in
feces) is negative. Stool culture detected no the lungs. Systolic murmur is detected over
pathogenic germs. What is the most likely the cardiac apex, heart rate is 100/min., BP
cause of diarrhea in this case? is 100/70 mm Hg. The epigastrium is slightly
tender on palpation. Blood test: erythrocytes
A. Antibiotic-associated diarrhea - 3.2 · 1012 /L, Нb- 100 g/L, color index -
B. Intestinal dysbiosis 0.95. What type of anemia is the most likely
C. Bacterial overgrowth syndrome present in this case?
D. Ulcerative colitis
E. Crohn’s disease (regional enteritis) A. Posthemorrhagic anemia
B. Sideroblastic anemia
4. A chronic alcoholic was hospitalized C. Chronic iron-deficiency anemia
into the therapeutic inpatient unit due D. Hemolytic anemia
to pneumonia. On the day 5 of his E. Hypoplastic anemia
hospitalization he became disoriented in
time and space, developed fear-inducing 8. A 62-year-old patient has been
visual hallucinations and motor agitation. hospitalized with complaints of pain in
Full body tremor and tremor of the limbs are the thorax on the right during breathing,
observed. X-ray and physical examinations dyspnea, and dry cough. Ten days ago he
detect the signs of his convalescence from slipped and fell hitting his right side. On
pneumonia. What tactics should be chosen examintaion: the patient lies on the left
regarding this patient? side. The right side of the thorax lags during
breathing. On the right there are crepitation
and pain in the III-IV ribs. Dullness of
percussion sound and sharply diminished
breath sounds can be observed. On X-ray:
Терапевтичний профiль 3

signs of exudate, fracture of the III-IV ribs. A. Acute left ventricular failure
On pleurocentesis: blood is detected. Choose B. Paroxysmal tachycardia
the further tactics: C. Bronchial asthma attack
D. Pulmonary embolism
A. Transfer to a thoracic surgery department E. Acute right ventricular failure
B. Prescribe conservative therapy
C. Perform repeated pleural taps 12. A 26-year-old woman is suspected to
D. Apply a fixation bandage to the rib cage suffer from systemic lupus erythematosus
E. Refer to a traumatologist due to systemic lesions of skin, vessels, joints,
serous tunics, and heart that developed
9. A 51-year-old man complains of vomiting after photosensitization. The following is
with blood. He has been drinking alcohol detected: LE cells, antibodies to native
excessively. Health disorder has been DNA, isolated anti-centromere antibodies,
observed since he was 40, when he first rheumatoid factor is 1:100, Wassermann
developed jaundice. On examination the reaction is positive, circulating immune
skin and visible mucosa are icteric, with complex is 120 units. What immunological
a stellate vascular pattern. The patient is indicators are considered to be specific to
malnourished and presents with abdominal this disease?
distension, umbilical hernia, and ascites. The
edge of the liver is tapered and painless, +3 A. DNA antibodies
cm, the spleen is +2 cm. Blood test: Hb- 80 B. Rheumatoid factor
g/L, leukocytes - 3·109 /L, platelets - 85·109 /L. C. Anti-centromere antibodies
What is the cause of portal hypertension in D. Immunoglobulin A
this patient? E. Increased circulating immune complex

A. Hepatic cirrhosis 13. A woman came to the doctor with


B. Thrombosis of the splenic vein complaints of increased body temperature up
C. Hemochromatosis to 37.8o C and moderately sore throat for the
D. Constrictive pericarditis last 3 days. Objectively: mandibular lymph
E. Budd-Chiari syndrome nodes are enlarged up to 3 cm. Palatine
tonsils are hypertrophied and covered with
10. A young man has made an appointment gray coating that spreads to the uvula and
with the dermatologist. He complains of a anterior pillars of the fauces. What is the
painful facial rash in the beard and mustache most likely diagnosis?
area. This condition has been persisting
for several weaks already. After shaving, A. Oropharyngeal diphtheria
the patient’s condition aggravates. The B. Infectious mononucleosis
diagnosis of sycosis is made. What primary C. Pseudomembranous (Vincent’s) tonsillitis
morphological elements can be observed in D. Agranulocytosis
the rash in this case? E. Oropharyngeal candidiasis

A. Pustules, papulae 14. A 42-year-old man, a dispatcher, suffes


B. Nodes, nodules from peptic ulcer disease of the duodenum.
C. Pustules, bumps The disease is of moderate severity. He wants
D. Phlyctenae, maculae to be assigned a disability group. Make the
E. Maculae, nodes conclusion regarding his working ability:

11. At night a 63-year-old woman suddenly A. Capable of working, employable


developed an asphyxia attack. She has a 15- B. Capable of working, non-employable
year-long history of essential hypertension C. First group of disability
and had a myocardial infarction 2 years ago. D. Second group of disability
Objectively her position in bed is orthopneic, E. Third group of disability
the skin is pale, the patient is covered with 15. A 58-year-old man complains of
cold sweat, acrocyanosis is observed. Pulse weakness and tumor-like formations that
- 104/min. Blood pressure - 210/130 mm appeared on the anterior surface of his neck
Hg, respiration rate - 38/min. Pulmonary and in the inguinal region. Palpation detects
percussion sound is clear, with slight dullness soft painless mobile cervical and inguinal
in the lower segments; throughout the lymph nodes up to 2 cm in diameter. The
lungs single dry crackles can be heard that liver protrudes by 2 cm from the edge of the
become bubbling and non-resonant in the costal margin, the lower splenic pole is at
lower segments. What is the most likely the umbilical level. In blood: erythrocytes
complication in this patient?
- 3.5 · 1012 /L, Hb- 88 g/L, leukocytes -
86 · 109 /L, band neutrophils - 1%, segmented
neutrophils - 10%, lymphocytes - 85%,
eosinophils - 2%, basocytes - 0%, monocytes
- 2%, erythrocyte sedimentation rate - 15
Терапевтичний профiль 4

mm/hour, Gumprecht shadows. What is the has been lasting for 2 years. Objectively:
most likely diagnosis? reduced spinal mobility, painful sacroiliac
joint, erythrocyte sedimentation rate - 45
A. Chronic lymphatic leukemia mm/hour. X-ray shows narrowing of the
B. Lymphocytic leukemoid reaction intervertebral disc space and of the sacroiliac
C. Acute leukemia joint. What eye pathology is often associated
D. Chronic myeloleukemia with this type of disease progression?
E. Lymphogranulomatosis
A. Iridocyclitis
16. A 23-year-old man complains of facial B. Retinal detachment
edemas, headache, dizziness, low urinary C. Cataract
output, and urine discoloration (dark red). D. Optic nerve atrophy
These complaints arose after a case of acute E. Blepharitis
tonsillitis. On examination there are facial
edemas, the skin is pale, temperature is 20. A 63-year-old man complains of
37.4o C ; heart rate is 86/min., blood pressure unmotivated weakness and pressing and
is 170/110 mm Hg. Heart sounds are muffled, bursting sensation in the left subcostal
the II heart sound is accentuated over the area. According to him, these signs
aorta. What etiological factor is the most have been present for a year already.
likely in this case? Previously he was healthy. He took part in
containment measures during the accident
A. Beta-hemolytic streptococcus at the Chornobyl Nuclear Power Plant.
B. Staphylococcus aureus Objectively: the skin is pale, peripheral
C. Streptococcus viridans lymph nodes are not enlarged, the liver
D. Streptococcus pyogenes is +3 cm, the spleen is +10 cm. Complete
E. Staphylococcus saprophyticus blood count: erythrocytes - 3.1 · 1012 /L,
17. An 18-year-old young man complains of Hb- 100 g/L, leukocytes - 46 · 109 /L, blasts
pain in his knee and elbow joints and body - 2%, promyelocytes - 10%, myelocytes -
temperature up to 39.5o C . One week and 18%, band neutrophils - 27%, segmented
a half earlier he developed sore throat. On neutrophils - 10%, lymphocytes - 12%,
examination his body temperature is 38.5o C . eosinophils - 6%, basocytes - 3%, monocytes
Swelling of the knee and elbow joints is - 2%, erythrocyte sedimentation rate - 20
observed. Pulse is 106/min., rhythmic. Blood mm/hour. What is the most likely diagnosis?
pressure is 90/60 mm Hg. Cardiac borders A. Chronic myeloleukemia
are unchanged, heart sounds are weakened, B. Hepatic cirrhosis
at the cardiac apex there is a soft systolic C. Acute leukemia
murmur. What factor would be the most D. Hemolytic anemia
indicative of the likely disease etiology? E. Chronic lymphatic leukemia
A. Anti-streptolysin O 21. For three years a 31-year-old woman has
B. C-reactive protein been complaining of pain and swelling of
C. Creatine kinase her radiocarpal and metacarpophalangeal
D. Rheumatoid factor articulations and their reduced mobility in
E. Seromucoid the morning, which persisted up to 1.5 hours.
18. A woman has been provisionally Two weeks ago she developed pain, swelling,
diagnosed with pheochromocytoma. At the and reddening of her knee joints, her body
stage of intermission her BP is within norm; temperature increased up to 37.5o C . The
there is a tendency towards tachycardia. No treatment was untimely. Examination of
urine pathologies. The decision has been the internal organs revealed no pathologic
made to perform a provocative test with alterations. Diagnosis of rheumatoid arthritis
histamine. What drug should be kept close was made. What changes are most likely to
at hand for emergency aid in case of positive be visible on the arthrogram?
test result? A. Joint space narrowing, usuration
A. Phentolamine B. Joint space narrowing, subchondral
B. Pipolphen (Promethazine) osteosclerosis
C. Nifedipine C. Cysts in the subchondral bone
D. Numerous marginal osteophytes
D. Mesaton (Phenylephrine) E. Epiphyseal osteolysis
E. Prednisolone
22. A 52-year-old woman has been
19. A 40-year-old man with Bekhterev suffering for 2 years from dull, occasionally
disease (ankylosing spondylitis) complains of exacerbating pain in her right subcostal
elevated body temperature up to 37.8o C , back area, occurring after eating high-fat foods,
pain and stiffness, especially observed during bitter taste in her mouth in the morning,
the second half of the night. This condition constipations, and flatulence. Objectively she
Терапевтичний профiль 5

has excess weight, her body temperature muscles. What is the most likely diagnosis?
is 36.9o C ; there is a coating on the root
of her tongue; the abdomen is moderately A. Trigeminal neuralgia
distended and painful in the area of B. Glossopharyngeal neuralgia
gallbladder projection. What examination C. Temporomandibular joint arthritis
would be the most helpful for diagnosis- D. Facial migraine
making? E. Maxillary sinusitis
A. Ultrasound 26. A 28-year-old man complains of skin
B. Duodenal intubation rash and itching on the both of his hands.
C. Cholecystography The condition persists for 1.5 years. The
D. Duodenoscopy exacerbation of his condition he ascribes to
E. Liver scanning the occupational contact with formaldehyde
resins. Objectively the lesion foci are
23. A 57-year-old woman complains of symmetrically localized on both hands.
weakness, dyspnea, loss of appetite, and Against the background of erythema with
liquid feces. She has been suffering from blurred margins there are papulae, vesicles,
this condition for 2 years. Objectively erosions, crusts, and scales. What is the most
she presents with pale skin, subicteric likely pathology?
sclerae, and bright-red fissured tongue.
Lymph nodes are not enlarged. Pulse A. Occupational eczema
- 100/min. BP- 105/70 mm Hg. Liver B. Idiopathic eczema
+3 cm, the spleen cannot be palpated. C. Allergic dermatitis
Blood test: erythrocytes - 1.2 · 1012 /L, D. Simple contact dermatitis
Нb- 56 g/L, color index - 1.4, macrocytes, E. Erythema multiforme
leukocytes - 2, 5 · 109 /L, eosinophils - 1%, 27. A 20-year-old student after failing an
juvenile - 1%, metamyelocytes - 1%, band exam developed complaints of a sensation of
neutrophils - 8%, segmented neutrophils - a round foreign body in her throat, difficult
47%, lymphocytes - 38%, monocytes - 4%, swallowing. She fixates on her condition,
reticulocytes - 0.1%, platelets - 100 · 109 /L, limits her diet, often cries, seeks attention,
ESR- 30 mm/hour, indirect bilirubin - 26 exhibits demonstrative attitude. She is highly
mmol/L. What changes can be expected in susceptible to psychotherapeutic suggestion.
the bone marrow puncture material? What psychiatric diagnosis can be made in
this case?
A. Prevalence of megaloblasts
B. Increased number of sideroblasts A. Hysterical neurosis
C. Erythroid hyperplasia B. Hypochondriacal neurosis
D. Presence of blast cells C. Depressive neurosis
E. Prevalence of lymphoid tissue D. Obsessive neurosis
E. Paranoid personality disorder
24. A 35-year-old man suffers from insulin-
dependent diabetes mellitus and chronic 28. A woman with atopic bronchial asthma
cholecystitis. He takes NPH insulin: 20 units was found to have one allergen to dog
in the morning and 12 units in the evening. hair +++. Carpets were removed from the
After a meal he developed pain in the right apartment, the apartment was renovated,
subcostal area, nausea, vomiting, sleepiness, and air conditioner was installed. However,
and increased polyuria. What prehospital recurrent asphyxia attacks still occur every
measures will be the most effective for night, despite the patient undergoing
prevention of crisis within the next several pathogenetic therapy. What long-term
hours? treatment tactics can help this patient to
decrease her sensitivity to the allergen?
A. Change insulin regimen
B. Take analgesics A. Specific hyposensitization
C. Take cholagogues B. Continuation of prior treatment
D. Exclude fats from the diet C. Antihistamine therapy
E. Decrease carbohydrates in the diet D. Buteyko breathing technique
E. Referral for speleotherapy
25. A 45-year-old woman complains of
intolerable paroxysmal facial pain on the left 29. A 20-year-old man was hospitalized on
with attacks that last for 1-2 minutes. Attacks the 9th day of the disease. He attributes
are provoked by chewing. The disease onset his disease to eating of insufficiently
was two months ago after the overexposure thermally processed pork. At its onset
to cold. Objectively: pain at the exit points this condition manifested as periorbital
of the trigeminal nerve on the left. Touching edemas and fever. Objectively his body
near the wing of the nose on the left induces temperature is 38.5 C . The face is puffy and
a pain attack with tonic spasm of the facial the eyelids are markedly swollen. Palpation
Терапевтичний профiль 6

of gastrocnemius muscles is sharply painful. exercise test at 50 W there is a depression


Blood test shows hypereosinophilia. What is of S-T segment by 3 mm below the isoline in
the etiology of this disease? V3-V4. What is the provisional diagnosis?
A. Trichinella A. Exertional angina pectoris, functional class
B. Trichuris III
C. Ascarididae B. Exertional angina pectoris, functional class
D. Echinococci IV
E. Leptospira C. Exertional angina pectoris, functional class
II
30. A 40-year-old man claims that his wife D. Somatoform autonomic dysfunction,
is cheating on him and presents a ”proof” hypertension type
of her infidelity. He repeatedly initiated E. Alcoholic myocardiodystrophy
scandals with his wife at home and at work,
demanding that she confess her infidelity, 34. A 45-year-old man, a farmer, presents
insulted her, and threatened to kill her. What with acute onset of a disease. He complains
preventive measures should be taken against of headache, high temperature, pain in the
socially dangerous actions on his part? gastrocnemius muscles, icteric face, and dark
urine. Objectively: body temperature - 38o C ,
A. Consultation with the psychiatrist blood pressure - 100/70 mm Hg, conjunctival
B. Outpatient treatment hemorrhages, hepatosplenomegaly, and
C. Consultation with the general practitioner oliguria. What is the most likely provisional
D. Consultation with the psychologist diagnosis?
E. Family counseling
A. Leptospirosis
31. A 55-year-old woman complains of B. Brucellosis
pain and popping sounds in her left knee C. Viral hepatitis
joint, which occur when she climbs the D. Pseudotuberculosis
stairs. Occasionally during movements her E. Trichinosis
joint becomes ”stuck”. 5 years ago she
suffered a trauma of her left knee. Complete 35. A woman has been working as a polisher
blood count and biochemical blood analysis for a year and a half. Her workstation
show normal results. X-ray shows marked is equipped with a grinding machine
osteosclerosis and osteophytes. The joint (grinding wheels). She complains of white
space is narrowed. Make the provisional discoloration of her fingers and toes that
diagnosis: appears when she is nervous. Objectively
there are no changes in the coloration of the
A. Osteoarthritis distal segments of her limbs. Grip strength
B. Rheumatoid arthritis measured with a dynamometer is 25 kg,
C. Gouty arthritis algesimetry findings are 0.1; 0.3; 0.5. Cold
D. Psoriatic arthritis stimulus is extremely positive on the upper
E. Reactive arthritis and lower limbs. Internal organs are without
pathologies. Make the diagnosis:
32. A 40-year-old man, a welder, uses
manganese electrodes in his line of work A. Vibration disease
(18 years of experience). He complains of B. Raynaud disease
difficulties with writing, bad mood, inertness, C. Syringomyelia
gait abnormalities, problems with speech, D. Raynaud syndrome
and hand tremors. Objectively the following E. Polyneuritis
is observed in the patient: hypomimia,
increased muscle tone of plastic type, and 36. A 37-year-old man suddenly developed
quiet monotonous speech, tremor of the acute headache accompanied by nausea,
tongue, pill-rolling tremor of the fingers, and vomiting, and impaired consciousness.
retropulsion. What syndrome developed in Objectively blood pressure is 190/120
this patient due to manganese poisoning? mm Hg, the face is hyperemic. Patient’s
consciousness is clouded, his answers to
A. Parkinsonism the questions are short, monosyllabic.
B. Meningism Movement and sensory disturbances are
C. Hypothalamic syndrome absent. Meningeal signs are positive.
D. Polyneuritic syndrome Cerebrospinal fluid contains blood. What
E. Vestibular syndrome provisional diagnosis can be made?
33. A 45-year-old man developed
constricting retrosternal pain that occurs
during walks at the distance of 200 m.
Objectively: heart rate is 80/min., BP is
160/90 mm Hg. During cardiopulmonary
Терапевтичний профiль 7

A. Subarachnoid hemorrhage A. Leukocyturia, gross hematuria


B. Meningitis B. Gross hematuria
C. Ischemic stroke C. Increased blood creatinine and blood urea
D. Encephalitis D. Daily proteinuria under 3.0
E. Cerebral vascular embolism E. Daily proteinuria over 3.0
37. A woman undergoing in-patient 41. A 62-year-old woman was brought
treatment for viral hepatitis type B into the admission room with complaints
developed headache, nausea, recurrent of severe burning retrosternal pain and
vomiting, memory lapses, flapping tremor asphyxia. She has a 10-year-long history
of her hands, and rapid pulse. Sweet of essential hypertension. Objectively her
smell from her mouth is detected. Body condition is moderately severe. She presents
temperature is 37.6o C , heart rate is 89/min. with skin pallor, cyanotic lips, and vesicular
What complication developed in the patient? respiration over her lungs. The II heart sound
is accentuated over the aorta. Blood pressure
A. Acute liver failure - 210/120 mm Hg, heart rate (pulse) - 76/min.
B. Ischemic stroke ECG shows elevation of ST segment in the
C. Gastrointestinal hemorrhage leads I, AVL, and V5-V6. What is the most
D. Hypoglycemic shock likely diagnosis?
E. Meningoencephalitis
A. Hypertensive crisis complicated with acute
38. A 43-year-old man, a coal-face worker myocardial infarction
with 15-year-long record of work, complains B. Uncomplicated hypertensive crisis
of cough, thoracic pain, and dyspnea. The C. Hypertensive crisis complicated with
cough is mild, usually dry, occurs mostly instable angina pectoris
in the morning. The pain is localized in D. Hypertensive crisis complicated with acute
the interscapular region and aggravates left ventricular failure
during a deep intake of breath. Dyspnea E. Pulmonary embolism
occurs during physical exertion. Vesicular
respiration in the lungs is weakened. Heart 42. A 35-year-old patient developed an
sounds are rhythmic, heart rate is 86/min., epileptic attack with tonoclonic spasms that
blood pressure is 135/80 mm Hg. The lasted for 3 minutes. After the attack the
abdomen is soft and painless. X-ray shows patient fell asleep but in 5 minutes the second
micronodular pulmonary fibrosis. Make the attack occurred. The first step of emergency
provisional diagnosis: aid would be to:
A. Carboconiosis A. Ensure patency of airways
B. Byssinosis B. Take blood from the vein for analysis
C. Siderosis C. Introduce diazepam intravenously
D. Berylliosis D. Prescribe antiepileptic drugs
E. Metal pneumoconiosis E. Administer chloral hydrate via an enema
39. Having examined a 52-year-old patient, 43. A 27-year-old woman, a teacher in the
the doctor diagnosed him with obesity (body elementary school, complains of frequent
mass index - 34 kg/m2 , waist circumference stools, up to 3 times per day, with lumpy
- 112 cm) and arterial hypertension (170/105 feces and large amount of mucus, abdominal
mm Hg). 2-hour postprandial blood sugar pain that gradually abates after a defecation,
is 10.8 mmol/L. What biochemical blood irritability. Her skin is pale and icteric. Pulse
analysis needs to be conducted to diagnose is 74/min., rhythmic, can be characterized
the patient with metabolic syndrome X? as satisfactory. Blood pressure is 115/70 mm
Hg. The abdomen is soft, moderately tender
A. Lipid profile along the colon on palpation. Fiberoptic
B. Bilirubin colonoscopy detects no changes. What
C. Calcium and phosphorus disease can be suspected?
D. Creatinine and urea
E. Electrolytes A. Irritable bowel syndrome
B. Chronic non-ulcerative colitis
40. After overexposure to cold a 45- C. Chronic enteritis
year-old woman developed acute pain in D. Crohn disease (regional enteritis)
her suprapubic and lumbar areas during E. Whipple disease
urination, sharp pains at the end of urination,
false urges to urinate. Urine is turbid with 44. A 72-year-old man complains of lower
blood streaks. The doctor suspects urinary extremity edema, sensation of heaviness in
tract infection. What results of laboratory the right subcostal area, dyspnea at rest.
analysis would be the most indicative of such For over 25 years he has been suffering
infection? from COPD. Objectively: orthopnea,
jugular venous distention, diffuse cyanosis,
Терапевтичний профiль 8

acrocyanosis. Barrel chest is observed, on treatment tactics:


percussion there is a vesiculotympanitic
(bandbox) resonance, sharply weakened A. Pericardial puncture (pericardiocenthesis)
vesicular respiration on both sides, moist B. Diuretics
crepitant crackles in the lower segments C. Antibiotics
of the lungs. Heart sounds are weakened, D. Pericardectomy
the II heart sound is accentuated over the E. Glucocorticosteroids
pulmonary artery. The liver is +3 cm. What
complicated the clinical course of COPD in 48. A 39-year-old man suffers from chronic
this patient? rheumatic heart disease. He complains of
dyspnea during physical exertion, cough with
A. Chronic pulmonary heart expectoration, and palpitations. Ausculation
B. Pulmonary embolism detects intensified I heart sound and diastolic
C. Acute left ventricular failure murmur; the sound of opening mitral valve
D. Diffuse pneumosclerosis can be auscultated at the cardiac apex.
E. Community-acquired pneumonia The II heart sound is accentuated over the
pulmonary artery. The patient is cyanotic.
45. A 72-year-old man with pneumonia X-ray shows dilated pulmonary root and
complains of marked dyspnea, chest pain, enlargement of the right ventricle and left
severe cough with expectoration, to is atrium. What is the most likely diagnosis?
39.5-40o C , no urination for a whole
day. Objectively the patient is conscious. A. Mitral stenosis
Respiratory rate is 36/min. Over the right B. Aortic stenosis
lower pulmonary lobe percussion sound C. Pulmonary artery stenosis
is dull; on auscultation there is bronchial D. Coarctation of the aorta
respiration and numerous moist crackles. E. Patent ductus arteriosus
Blood pressure is 80/60 mm Hg. Heart rate is
120/min. Heart sounds are muffled, there is 49. A 23-year-old man complains of severe
tachycardia. What tactics should the family pain in his left knee joint. Objectively the
doctor choose in the management of this left knee joint is enlarged, with hyperemic
patient? skin, painful on palpation. Complete blood
count: erythrocytes - 3.8 · 1012 /L, Hb- 122
A. Hospitalization into the intensive care unit g/L, leukocytes - 7.4 · 109 /L, platelets -
B. Outpatient treatment 183 · 109 /L. Erythrocyte sedimentation rate
C. Treatment in the day patient facility - 10 mm/hour. Bleeding time (Duke method)
D. Hospitalization into the pulmonology unit - 4 min., Lee-White coagulation time - 24
E. Hospitalization into the neurology unit min. Partial thromboplastin time (activated)
46. 2 hours after eating unknown - 89 seconds. Rheumatoid factor - negative.
mushrooms, a 28-year-old man sensed a What is the most likely diagnosis?
decrease in his mobility and deterioration of A. Hemophilia, hemarthrosis
his ability to focus. This condition was then B. Werlhof disease (immune
followed by a state of agitation and agression.
On examiantion he is disoriented and his thrombocytopenia)
speech is illegible. 4 hours later he developed C. Rheumatoid arthritis
fetor hepaticus and lost his consciousness. D. Thrombocytopathy
What syndrome can be observed in this E. Hemorrhagic vasculitis (Henoch-Schonlein
patient? purpura), articular form

A. Acute hepatic failure 50. A 24-year-old woman, a kindergarten


B. Hepatolienal syndrome teacher, has been sick for 2 days already.
C. Portal hypertension Disease onset was acute. She presents with
D. Cholestatic syndrome elevated body temperature up to 38.0o C ,
E. Cytolytic syndrome pain attacks in her lower left abdomen, liquid
stool in small amounts with blood and mucus
47. A 36-year-old man complains of marked admixtures 10 times a day. Pulse - 98/min.,
dyspnea and cardiac pain. He ascribes his blood pressure - 110/70 mm Hg. Her tongue
disease to the case of influenza that he had is moist and coated with white deposits. The
2 weeks ago. Objectively he leans forward abdomen is soft, the sigmoid colon is painful
when sitting. The face is swollen, cyanotic, and spastic. Make the provisional diagnosis:
cervical veins are distended. Heart borders
are extended on the both sides, heart sounds A. Shigellosis
are muffled, heart rate = Ps = 118/min., B. Escherichiosis
BP is 90/60 mm Hg. Blood test: ESR is C. Salmonellosis
46 mm/hour. ECG shows low voltage. X- D. Yersiniosis
ray shows trapezoidal cardiac silhouette and E. Rotavirus infection
signs of pulmonary congestion. Choose the
Терапевтичний профiль 9

51. A 38-year-old woman complains of tuberculosis clinic. Throughout the last 3


weakness, sleepiness, pain in the joints, weeks he has been suffering from headaches
weight gain despite low appetite, and of increasing intensity. Neurological
constipations. She presents with dry and examination detects nuchal rigidity without
thickened skin, puffy and amimic face, focal signs. Make the provisional diagnosis:
narrowed palpebral fissures, thick tongue,
and deep hoarse voice. Her heart sounds are A. Tuberculous meningitis
weak, pulse is 56/min. Low levels of free T4 B. Chorea minor
are observed. This patient needs to take the C. Brain tumor
following on a regular basis: D. Myelitis
E. Convexital arachnoiditis
A. Thyroxine
B. Mercazolil (Thiamazole) 56. A patient has gradually lost
C. Lithium carbonate consciousness. The skin is pale and dry. There
D. Furosemide is a smell of ammonia from the mouth.
E. Calcium gluconate Respirations are deep and noisy. Heart
sounds are muffled, pericardial friction rub
52. A 23-year-old man has accidentally is present. Blood pressure is 180/130 mm
swallowed brake fluid. After that he has been Hg. Blood test: Нb- 80 g/L, leukocytes -
presenting with anuria for 5 days already; 12 · 109 /L, blood glucose - 6.4 mmol/L, urea -
his creatinine levels elevated up to 0.569 50 mmol/L, creatinine - 1200 mcmol/L, blood
mmol/L. What treatment tactics should be osmolarity - 350 mOsmol/L. No urinary
chosen in this case? excretion. Make the diagnosis:
A. Hemodialysis A. Uremic coma
B. Detoxication therapy B. Hyperglycemic coma
C. Antidotal therapy C. Acute renal failure
D. Diuretics D. Acute disturbance of cerebral circulation
E. Plasmapheresis E. Hyperosmolar coma
53. A 52-year-old man for the last 3 years 57. A 72-year-old man diagnosed with
has been suffering from difficult swallowing ischemic heart disease presents with diffuse
of solid food, burning retrosternal pain cardiosclerosis, permanent tachysystolic
that aggravated during eating, loss of atrial fibrillation, heart failure IIа, FC III.
body mass, and occasional vomiting with Objective examination of vital signs: blood
undigested food. Esophageal X-ray shows pressure is 135/80 mm Hg, heart rate is
S-shaped deformation of the esophagus 160/min., pulse is 125/min. Left ventricular
and its dilation; at the cardiac orifice the ejection fraction is 32%. What drug is
esophagus is constricted; esophageal mucosa indicated in this case and should be presribed
is smooth, without signs of peristalsis. Make to the patient?
the provisional diagnosis:
A. Digoxin
A. Esophageal carcinoma B. Procainamide (Novocainamide)
B. Diaphragmatic hernia C. Isadrine (Isoprenaline)
C. Esophageal achalasia D. Verapamil
D. Reflux esophagitis E. Ivabradine
E. Esophageal diverticulum
58. A 34-year-old man complains of pale
54. A 53-year-old man complains of edema of the face, feet, shins, and lumbar
general weakness, loss of appetite, and area, elevated blood pressure up to 160/100
painful vesicles appearing on his skin. mm Hg, and general weakness. He has
The disease onset occurred suddenly, after a clinical history of nonspecific ulcerative
hyperinsolation one week ago. Examination colitis. Objectively: pulse - 84/min., rhythmic,
detects isolated vesicles with wrinkled blood pressure - 165/100 mm Hg; edemas all
opercula and occasional painful erosions on over the body; the skin is pale and dry, with
the skin of the patient’s torso and limbs. low turgor. The kidneys cannot be palpated,
Nikolsky sign is positive. What is the most on an attempt to palpate them they are
likely diagnosis? painless. Blood test: erythrocytes - 3.0·1012 /L,
A. Acantholytic pemphigus Нb- 100 g/L, erythrocyte sedimentation
B. Nonacantholytic pemphigus rate - 50 mm/hour. Urinalysis: proteins -
3.5 g/L, erythrocytes - 7-10 in the vision
C. Duhring’s disease (dermatitis field, leukocytes - 5-6 in the vision field.
herpetiformis) Daily proteinuria - 6 grams. What analysis
D. Herpes should be conducted additionally to verify
E. Toxicodermia the diagnosis?
55. A patient is being treated in the
Терапевтичний профiль 10

A. Gingival biopsy for the diagnosis of blood circulation. Oblectively her pulse is
amyloid disease 96/min., of unequal strength. Blood pressure
B. Radioisotopic examination of kidneys is 110/70 mm Hg, heart rate is 120/min. ECG
C. Urinalysis for Bence-Jones protein registers small unevenly-sized waves in place
D. Renal ultrasound of P-waves, R-R intervals are of unequal
E. Survey and excretory urography length. What is the most likely diagnosis?
59. A 42-year-old man, a worker at the A. Atrial fibrillation
meat processing factory, developed an B. Paroxysmal supraventricular tachycardia
itching spot on his lower jaw, which C. Atrial flutter
gradually transformed into a slightly painful D. Paroxysmal ventricular tachycardia
carbuncle 3 cm in diameter, surrounded by E. Respiratory arrhythmia
a painless swelling that reaches the clavicle.
Temperature is subfebrile, under 37.8o C . The 63. An 18-year-old patient always obeys
doctor suspects anthrax. What drug should others and adapts his needs to the demands
this man be prescribed for treatment? of the people on whom he depends. He
excessively defers to their wishes and makes
A. Penicillin them responsible for his wellbeing, cannot
B. Levomycetin (Chloramphenicol) defend his interests and needs support
C. Biseptol (Co-trimoxazole) from other people. Such psychic profile
D. Interferon alpha has been formed in the childhood, remains
E. Azidothymidin (Zidovudine) unchanged, and hinders adaptation. What
psychic disorder is observed in this patient?
60. A 57-year-old patient complains of
dyspnea at rest. The patient presents with A. Dependent personality disorder
orthopnea, acrocyanosis, bulging cervical B. Anxiety (avoidant) personality disorder
veins. On percussion: dull sound over the C. Anankastic personality disorder
lower lung segments. On auscultation: no D. Markedly accentuated personality
respiratory sounds. Heart rate is 92/min. E. Psychopathy-like state
Right-sided cardiac dilatation is observed.
The liver is +7 cm. Shins are swollen. Pleural 64. A 45-year-old man with thrombophlebitis
effusion is suspected. What indicator would of the deep veins in his legs suddenly after
confirm the presence of transudate in this physical exertion developed sharp pain in his
case? thorax on the right, dyspnea, and hemoptysis.
Objectively his condition is severe; he
A. Total protein content in the pleural fluid presents with acrocyanosis, shortening of
below 25 g/L pulmonary percussion sound on the right,
B. Presence of atypical cells and weakened respiration. Respiration is
C. Total protein content in the pleural fluid 30/min., blood pressure is 110/80 mm Hg.
exceeding 30 g/L ECG shows sinus tachycardia, heart rate is
D. Specific gravity exceeding 1015 120/min., electrical axis of the heart deviates
E. Positive Rivalta’s test to the right, SI -QIII . What is the most likely
diagnosis?
61. A 33-year-old man developed multiple
rashes on the skin of his torso and extensor A. Pulmonary embolism
surfaces of his upper and lower limbs. B. Community-acquired right-sided
The rashes itch and occasionally fuse pneumonia
together and form plaques. The elements C. Cancer of the right lung
of rash are covered with silver-white fine D. Right-sided exudative pleurisy
scales that easily flake off when scratched. E. Spontaneous pneumothorax
Grattage test results in three sequential
phenomena: stearin spot, terminal film, and 65. A 48-year-old woman has been
punctate hemorrhage. What diagnosis can be hospitalized due to development of
suspected? tachysystolic atrial fibrillation. She has lost
5 kg of body weight within 2 months. On
A. Psoriasis palpation there is a node in the left lobe of
B. Parapsoriasis the thyroid gland. What pathology resulted
C. Pyoderma in the development of this condition?
D. Lichen ruber planus
E. Secondary papular syphilid A. Toxic nodular goiter
B. Aterosclerotic cardiosclerosis
62. A 38-year-old woman after physical C. Chronic thyroiditis
overexertion suddenly developed palpitations, D. Nontoxic nodular goiter
dyspnea, and a dull pain in the cardiac E. Autoimmune thyroiditis
area. For 10 years she has been registered
for regular check-ups due to rheumatism 66. A 48-year-old woman developed
and mitral valve disease with non-disturbed insomnia, depressive mood, anxiety, fears
Терапевтичний профiль 11

and suicidal thoughts after the death of A. β -blockers


her husband that occurred one month ago. B. Angiotensin-converting enzyme inhibitors
During her stay in the hospital she speaks C. Diuretics
in a low voice, is depressed, anxious, avoids D. Calcium antagonists
sleeping, refuses to eat. What medications E. Imidazoline receptor antagonists
should be prescribed in this case?
71. A 45-year-old woman is registered for
A. Antidepressants regular check-ups due to Werlhof disease
B. Antipsychotics (immune thrombocytopenia). Complete
C. Group B vitamins blood count: Нb- 100 g/L, erythrocytes -
D. Nootropics 2.8 · 1012 /L, platelets - 90.0 · 109 /L, leukocytes
E. Anticonvulsants - 8.4 · 109 /L, erythrocyte sedimentation rate -
13 mm/hour. Examination detects a single
67. A 32-year-old woman complains of small hematoma on the anterior surface
episodes of intense fear that occur without of the thigh, developed after the patient
visible cause and last for 10-20 minutes; the accidentally stumbled on a table. What
episodes are characterized by rapid pulse, treatment tactics should be chosen in this
sweating, labored breathing, and vertigo. case?
Specify the likely diagnosis:
A. Continue the supervision by the hospital
A. Panic disorder hematologist
B. Paranoid syndrome B. Urgent hospitalization into the hematology
C. Manic syndrome unit
D. Simple schizophrenia C. Urgently start a hemostatic therapy,
E. Claustrophobia followed by a planned hospitalization into the
68. A 39-year-old man suffers from hematology unit
chronic adrenal insufficiency and receives D. Urgent hospitalization into the general
replacement glucocorticoid therapy care unit
(hydrocortisone - 15 mg/day). He is to E. Administer thrombocytic mass, continue
undergo elective surgery for calculous the treatment in the hematology unit
cholecystitis. What medication adjustment 72. The dermatologist has an appointment
should be made on the day of the surgery with a 30-year-old man that complains
to prevent the development of acute adrenal of severely itching rashes that especially
insufficiency? disturb him at night. The rashes developed
A. Increase the dosage by 2-3 times 2 weeks ago, after he had returned from a
B. Cancel the drug for the day of the surgery travel. Objectively on the lateral surfaces
C. Add a mineralocorticoid of his fingers, hands, wrists, elbows, lower
D. Add an antibiotic abdomen, genitals, and thighs there are
E. Prescribe a large volume intravenous fluid paired papulovesicles, single pustules,
infusion and scratch marks. What disease can be
suspected?
69. After a long drive with the window
open a man developed facial asymmetry; A. Scabies
he cannot close his right eye, his right B. Pyoderma
nasolabial fold is smoothed out, movements C. Dermatitis
D. Eczema
of expression are absent on the right, there E. Shingles
is a disturbance of gustatory sensation in the
tongue on the right. No other neurological 73. A 38-year-old woman developed a
pathologies were detected. What disease can medical condition 7 days after her return
be provisionally diagnosed in this patient? from Bangladesh. Periodical elevation
A. Neuropathy of the facial nerve of temperature was accompanied by
B. Neuropathy of the trigeminal nerve chills and excessive sweating. She was
C. Trigeminal ganglionitis diagnosed with tropical malaria. Next day
D. Neuropathy of the oculomotor nerve her condition further deteriorated: body
E. Ischemic stroke temperature - 38o C , inertness, periodical
loss of consciousness, generalized seizures,
70. A 56-year-old woman was diagnosed with tachycardia, hypotension, and icteric skin.
stage 2 hypertension of the 2nd degree. She What complication can be suspected in this
belongs to the group of moderate risk and case?
has bronchial asthma. What group of drugs is
CONTRAINDICATED to this patient?
Терапевтичний профiль 12

A. Cerebral coma creatorrhea. What prescription would be the


B. Serous meningitis most advisable in this case?
C. Purulent meningitis
D. Acute hepatic failure A. Multi-enzyme preparations
E. Acute heart failure B. Cholinergic antagonists
C. Metronidazole and loperamide
74. A 73-year-old woman came to the family D. Antacids and antispasmodics
physician for one of her regular follow-up E. Cholinergic antagonists and antibacterial
examinations. Three months ago she was agents
found to have type 2 diabetes mellitus.
She was keeping to her diet and exercise 77. A man was brought into the admission
plan and taking phytopreparations. On room after an overexposure to cold. He
examination her fasting glucose was within complains of sharp pain in the small of
the range of 7.8-8.6 mmol/L, HbА1с - 7.9%. his back and elevated body temperature
Height - 164 cm, weight - 83 kg. What up to 38o C . He took some aspirin. Blood
blood sugar-controlling medicine should test: leukocytes - 10.5 · 1012 /L, eosinophils
she be prescribed first in the course of her - 5%, band neutrophils - 8%, segmented
pharmacological therapy? neutrophils - 51%, lymphocytes - 32%,
monocytes - 4%, erythrocyte sedimentation
A. Metformin rate - 28 mm/hour. Urinalysis: protein - 0.6
B. Glibenclamide g/L, leukocytes - cover the whole vision field,
C. Glimepiride large amount of mucus. What is the most
D. Gliclazide likely diagnosis?
E. Insulin
A. Acute pyelonephritis
75. A 27-year-old man complains of pain in B. Chronic pyelonephritis
his leg joints, purulent discharge from the C. Acute glomerulonephritis
eyes, and painful burning sensations during D. Tubulointerstitial nephritis
urination. Disease onset was acute. He has a E. Subacute malignant glomerulonephritis
history of influenza. The patient smokes and
drinks alcohol in excess. In his line of work 78. A 26-year-old man complains of
he is often away on business trips. What is the chills, rhinitis, dry cough, and fever up to
most likely etiological factor of this disease? 38o C . Examination shows him to be in
a moderately severe condition; there are
A. Chlamydia small pale pink non-merging spots on the
B. Adenovirus skin of his back, abdomen, and extremities.
C. Streptococci Palpation reveals enlarged occipital and
D. Staphylococci axillary lymph nodes. No information about
E. Candida vaccination history could be obtained. What
76. A 46-year-old woman has diarrhea with is the likely etiology of this disease?
abdominal distension, loss of body mass, A. Rubella virus
and large amounts of porridge-like foul- B. Epstein-Barr virus
smelling stool without blood streaks or C. Streptococcus
tenesmus. Objective examination detects D. Mumps virus
moderate tenderness in the mesogastrium E. Neisseria meningitis
and left abdominal flank. Feces analysis
detects steatorrhea with neutral fat and
Хiрургiчний профiль 13

1. During medical examination a cadet A. Frostbite of the I degree


in the naval college was detected to have B. Perniosis
a painless dense ulcer 1.5x0.5 in size in his C. Frostbite of the II degree
perianal area at the 2 o’clock position. The D. Frostbite of the III degree
ulcer floor resembles ”old fat”. What is the E. Frostbite of the IV degree
provisional diagnosis?
5. A 16-year-old patient has made an
A. Hard syphilitic chancre of the rectum appointment with an otolaryngologist. He
B. Rectal fissure complains of elevated body temperature
C. Rectal fistula and sore throat. Disease onset was 2 days
D. Anal cancer ago, after the patient ate two portions of
E. Anal crypt suppuration ice-cream. Pharyngoscopy shows hyperemic
mucosa of the palatine tonsils, with purulent
2. A 32-year-old woman complains of tumor- exudate in the lacunae. Make the provisional
like formation on the anterior surface of her diagnosis:
neck that appeared 2 years ago. Within the
last 3 months the tumor has been rapidly A. Lacunar tonsillitis
growing. It hinders swallowing and impairs B. Follicular tonsillitis
speech; the tumor causes a sensation of C. Diphtheria
pressure. Objectively the skin moisture is D. Acute pharyngitis
normal, pulse is 80/min., rhythmic, blood E. Pseudomembranous (Vincent’s) tonsillitis
pressure is 130/80 mm Hg. In the right lobe of
the thyroid gland there is a dense lumpy node 6. A 35-year-old woman complains of high
3.0x3.5 cm that moves during swallowing. body temperature and pain in the upper
Scanning image shows a ”cold nodule” in outer quadrant of her right buttock, which
the thyroid gland. Make the provisional developed after an injection. She has been
diagnosis: presenting with this condition for 3 days. At
the site of injection the skin is hyperemic;
A. Thyroid cancer there is a painful infiltrate with an area
B. Thyroid adenoma of softening in its center. The woman is
C. Thyroid cyst diagnosed with a postinjection abscess of
D. Nodular goiter the right buttock. What tactics should the
E. Autoimmune thyroiditis surgeon choose in this case?
3. After a surgery for a left thigh phlegmon A. Abscess incision, sanation and drainage of
the disease progression was complicated by the cavity
sepsis. On the 7th day after the surgery B. Hospitalization, prescription of antibiotics,
there are marked signs of a generalized UHF
inflammatory reaction, in blood there are C. Abscess puncture, pus removal followed by
signs of toxic anemia and progressing application of antiseptics
hypoproteinemia, bilirubin levels are 40 D. 10-15 minutes of low-intensity laser
mcmol/L, AST and ALT exceed the norm radiation directed at the right buttock
by 2.5 times. Oliguria persists (700 mL of E. Antipyretic agents, massage, and
urine per day). Name the phase of sepsis application of dry heat to the right buttock
progression:
7. A 65-year-old woman on abdominal
A. Catabolic phase palpation presents with a tumor in the
B. Stress phase umbilical region and above it; the tumor is
C. Anabolic phase 13x8 cm in size, moderately painful, non-
D. Recovery phase mobile, pulsing. On auscultation systolic
E. Mixed phase murmur can be observed. What is the most
likely diagnosis?
4. A 10-year-old boy, who was outdoors in
windy and cold weather, developed moderate A. Abdominal aortic aneurysm
pain and tingling in his fingers and toes. B. Gastric tumor
When he returned home, his parents noticed C. Arteriovenous aneurysm
that the tips of his fingers and toes were white D. Tricuspid insufficiency
and their sensitivity was lost. As the affected E. Bicuspid insufficiency
areas were warming up, the fingers and toes
developed tingling and painful sensations. 8. A 32-year-old man complains of pain
Skin pallor changed into redness, tingling in his legs that intensifies during walking,
stopped, mild itching and swelling of the intermittent claudication, numbness of his
fingers appeared. Determine the frostbite toes, extremity coldness, and inability to walk
degree in this child: more that 100 meters. When he sleeps, his
leg usually hangs down. The patient has been
smoking since he was 16. He drinks alcohol
in excess. The left leg is colder than the right
Хiрургiчний профiль 14

one; the skin of the extremities is dry. No burns of the 15% of the body surface. On the
pulse can be detected on the pedal arteries, 20th day after the trauma the patient presents
while pulsation of the femoral arteries is with sharp increase of body temperature,
retained. What is the most likely diagnosis? general weakness, rapid vesicular respiration;
facial features are sharpened, BP is 90/50 mm
A. Obliterating endarteritis Hg, heart rate is 112/min. What complication
B. Diabetic angiopathy is it?
C. Leriche syndrome (aortoiliac occlusive
disease) A. Sepsis
D. Raynaud disease B. Pneumonia
E. Deep thrombophlebitis C. Acute intoxication
D. Purulent bronchitis
9. A 50-year-old patient was brought to E. Anaerobic infection
a hospital with complaints of blood in
urine. Urination is painless and undisturbed. 13. 2 hours after a traffic accident a 28-year-
Macrohematuria had been observed for old man in a grave condition was brought
3 days. Objectively: kidneys cannot be to a hospital. The patient complains of
palpated, suprapubic area is without abdominal pain. He received a blow to the
alterations, external genitalia are non- abdomen with the steering wheel. Objective
pathologic. On rectal investigation: prostate examination revealed the following: the
is not enlarged, painless, has normal abdomen does not participate in respiration,
structure. Cystoscopy revealed no changes. is tense and acutely painful on palpation;
What is the most likely diagnosis? the abdominal muscles are defensively
tense, peritoneal irritation signs are positive,
A. Renal carcinoma hepatic dullness is absent. BP is 90/60 mm
B. Bladder tuberculosis Hg, heart rate is 120/min. What further
C. Varicocele treatment tactics should be chosen?
D. Dystopic kidney
E. Necrotic papillitis A. Laparotomy
B. Laparoscopy
10. A 59-year-old man complains of pain C. Cold to the abdomen
in his left eye and left side of his head, D. Ultrasound investigation
significant vision impairment of the left eye, E. Laparocentesis
nausea, and vomiting. Visual acuity of the
right eye is 1.0. Visual acuity of the left 14. A 48-year-old woman has arrived to
eye is 0.03, attempts at correction bring no the surgical unit with wounds in her thigh.
improvement. Right eye intraocular pressure On examination the wound surface has
- 21 mm Hg, left eye intraocular pressure - dirty-gray coating with unpleasant sweet
65 mm Hg. Congestive injection is observed smell. Wound content resembles raspberry
on the sclera of the left eye. The cornea is jelly. Skin tissues around the wound are
thick and swollen. The anterior chamber is glossy and turgid. Palpation reveals moderate
shallow, moist, and clear. The pupil is dilated crepitation in the tissues. What microflora is
and unresponsive to the light, the fundus of the most likely to cause such inflammation?
the eye is not visible. What is the most likely
diagnosis? A. Anaerobic clostridial
B. Anaerobic non-clostridial
A. Acute attack of glaucoma of the left eye C. Streptococci
B. Acute iridocyclitis of the left eye D. Staphylococci
C. Stage II intraocular tumor of the left eye E. Blue pus bacillus
D. Endophthalmitis of the left eye
E. Panophthalmitis of the left eye 15. After a pain attack in the right subcostal
area, a 58-year-old woman with overnutrition
11. On the 15th day after a small trauma developed icteric skin and sclera, light-
of the right foot, the patient developed colored feces, and dark urine. Her abdomen
indisposition, fatigability, irritability, is distended and painful on palpation in the
headache, elevated body temperature, and right subcostal area. Palpation detects liver
sensation of constriction, tension, and enlargement by 2-3 cm. Blood test: total
twitching in the muscles of the right shin. bilirubin - 90 mcmol/L, conjugated bilirubin
What disease can be suspected? - 60 mcmol/L. What method of examination
will be the most informative for diagnosis
A. Tetanus clarification?
B. Anaerobic gas gangrene
C. Erysipelas
D. Acute thrombophlebitis
E. Thrombophlebitis of the popliteal artery
12. A patient has the second and third degree
Хiрургiчний профiль 15

A. Retrograde cholangiopancreatography g/L, thrombocytopenia. Anamnesis states


B. Intravenous cholegraphy that a similar condition was observed 1 year
C. Infusion cholegraphy ago. Make the diagnosis:
D. Percutaneous transhepatic cholegraphy
E. US of the hepatopancreatobiliary zone A. Thrombocytopenic purpura
B. Hemophilia
16. An 11-year-old boy for a month has been C. Ulcerative bleeding
presenting with increasing pain in the right D. Rectal tumor
femur. In the painful area there is a non- E. Nonspecific ulcerative colitis
mobile painful tumor with unclear margins.
The child complains of general indisposition, 20. A 30-year-old man came to the family
weakness, increased body temperature up to physician. 2 months ago he underwent a
39o C . X-ray shows widened medullary cavity, surgery for open fracture of the humerus.
small foci of cancellous bone destruction, and On examination the patient’s condition is
onion-like lamellar exfoliation of the cortical satisfactory; in the area of the postoperative
layer. What is the most likely pathology wound there is a fistula that discharges
resulting in such clinical presentation? a small amount of pus; the area itself is
red; fluctuation is detected. X-ray shows
A. Ewing sarcoma destruction of the humerus with sequestra.
B. Osteogenic sarcoma What complication did the patient develop
C. Fibrosarcoma during the postoperative period?
D. Chondrosarcoma
E. Juxtacortical sarcoma A. Posttraumatic osteomyelitis
B. Hematogenous osteomyelitis
17. A 43-year-old man complains of a C. Wound suppuration
protrusion in the right inguinal region, D. Posttraumatic phlegmon
that enlarges due to strain. He has been E. Suture sinus
presenting with this condition for 6 months.
Within this period the protrusion has grown. 21. 3 hours after a trauma, a young
Objectively in the right inguinal region an man developed bradycardia of 46/min.,
elastic protrusion 8x5 cm is visible. On anisocoria D>S, hemi-hyperreflexia S>D,
palpation it disappears, leaving an empty hemihypesthesia on the left, and a convulsive
space 4x4 cm between the pedicles of disorder. The character of this process needs
the Poupart ligament. ”Cough push” sign to be clarified. What method of examination
is positive over this opening. Make the will be the most accurate for this purpose?
diagnosis:
A. Brain CT
A. Right-sided reducible inguinal hernia B. Skull X-ray
B. Right-sided reducible femoral hernia C. Electroencephalography
C. Cyst of the right spermatic cord D. Echoencephalography
D. Right-sided inguinal lymphadenitis E. Lumbar puncture
E. Right-sided reducible arcuate line hernia
22. The body of a 24-year-old woman with
18. A 78-year-old man with a prostate suspected poisoning has been found on
adenoma underwent a herniotomy for a the street. Forensic medical examination
direct inguinal hernia. After the surgery he was requested by an investigator during
presents with absent urination. Enlarged examination of the site and the body.
urinary bladder is detectable above the According to the Criminal Procedure Code
patient’s pubis. What measures should be currently in force in Ukraine, forensic
taken in this case? medical examination is required when it is
necessary to determine the:
A. Bladder catheterization
B. Apply cold to the urinary bladder area A. Cause of death
C. Prescribe processing of the postoperative B. Manner of death
wound with UHF field C. Time of death
D. Prescribe proserin (neostigmine) D. Mode of death
intramuscularly E. Mechanism of death
E. Prescribe antispasmodics subcutaneously
23. A 37-year-old patient complains of pain
19. A 38-year-old patient has been brought in the spinal column, reduced mobility. The
by an ambulance to the surgical department condition persists for 7 years. ”Sway back”
with complaints of general weakness, is observed, there is no movement in all
indisposition, black stool. On examination spinal regions. X-ray shows ”bamboo spine”
the patient is pale, there are dotted vertebral column. What is the most likely
hemorrhages on the skin of his torso and diagnosis?
extremities. On digital investigation there are
black feces on the glove. Blood test: Hb- 108
Хiрургiчний профiль 16

A. Ankylosing spondyloarthritis A. Acute appendicitis


B. Osteochondrosis B. Right-sided renal colic
C. Spondylitis deformans C. Cecal tumor
D. Tuberculous spondylitis D. Intestinal obstruction
E. Spondylolisthesis E. Acute cholecystitis
24. A surgery unit received a person with 28. A 45-year-old man diagnosed with acute
an incised stab wound on the upper third pulmonary abscess suddenly developed
of the right thigh. Examination detects an sharp pain in his chest on the right and
incised stab wound 3.0x0.5x2.0 cm in size dyspnea up to 30/min. Examination detects
on the inner surface of the upper third of facial cyanosis and shallow rapid respirations.
the right thigh. Bright-red blood flows from Auscultation reveals acutely weakened
deep within the wound in a pulsing stream. respiration throughout the whole right lung;
Characterize this type of bleeding: percussion reveals a vesiculotympanitic
(bandbox) resonance at the lung apex
A. Arterial and dullness in the lower lobe. What
B. Venous complication developed in this patient?
C. Parenchimatous
D. Capillary A. Pyopneumothorax
E. Mixed B. Pleuropneumonia
C. Pneumothorax
25. A 47-year-old man developed the D. Acute mediastinitis
signs of decompensated laryngeal stenosis E. Esophageal perforation
against the background of acute flegmonous
laryngitis. He presents with inspiratory 29. A 5-year-old child was brought to the
dyspnea at rest, forced position, cyanotic skin ENT department by an ambulance. The
covered in cold sweat, tachycardia, deficient child presents with cough and difficult
pulse, and low blood pressure. What urgent respiration. From the patient’s history it
treatment tactics should be chosen? is known that the child was playing with
a toy construction set, when suddenly
A. Tracheostomy started coughing and developed labored
B. Oral administration of hyposensitization breathing. Examination detects periodical
substances and broncholytics cough, labored expiration, and respiratory
C. Intravenous administration of dehydrating lag in the left side of the child’s thorax.
agents Auscultation: diminished respiration on the
D. Administration of glucocorticoid hormones left. Percussion: tympanitis. X-ray shows a
E. Oxygen therapy displacement of the mediastinal organs to
the right. Make the diagnosis:
26. Heart X-ray of a 31-year-old man has
revealed the following: with tightly filled A. A foreign body in the left bronchus,
opacified esophagus there is a marginal filling valvular bronchostenosis
defect in its middle third on the posterior B. A foreign body in the right bronchus,
wall; the defect is 1.8x1.3 cm in size with valvular bronchostenosis
clear oval border. Mucosal folds are retained C. A foreign body in the trachea
and envelop the defect; wall peristalsis D. A foreign body in the left bronchus,
and elasticity are not affected. There are complete bronchostenosis
no complaints regarding the condition of E. A foreign body in the right bronchus,
the patient’s alimentary canal. Make the partial bronchostenosis
provisional diagnosis:
30. A 30-year-old man was brought to the
A. Esophageal tumor neurosurgical department with complaints
B. Achalasia cardiae of constant headaches, nausea, vomiting,
C. Esophageal burns fever, and weakness of the right-side limbs.
D. Diverticulum Anamnesis states that one month ago
E. Barrett esophagus the patient had a surgery for left-sided
suppurative otitis and mastoiditis. He has
27. A 25-year-old man was hospitalized with been undergoing treatment in an ENT
complaints of pain in his lower abdomen and department. Approximately 2 weeks ago
right lumbar area that appeared one hour the temperature increased, and the patient
ago. Patient’s general state is moderately developed headaches. Objectively: heart rate
severe. Body temperature - 38.2o C , heart rate - 98/min., BP- 140/90 mm Hg, temperature -
- 102/min. The tongue is dry. The abdomen 38.3o C . Neurologically manifested stiff neck:
is painful on deep palpation in the right bilateral Kernig’s symptom, unsteadiness
iliac area and in the Petit triangle. Aure- during the Romberg’s maneuver. Computer
Rozanov and Gabay signs are positive. Make tomography of the brain revealed a three-
the provisional diagnosis: dimensional growth with a capsule in the left
hemisphere. Make the diagnosis:
Хiрургiчний профiль 17

sputum in the amount of 100 mL per


A. Cerebral abscess day, hemoptysis, dyspnea, increased body
B. Echinococcus temperature up to 37.8o C , general weakness,
C. Hemorrhage weight loss. The patient’s condition lasts for
D. Hydrocephalus 4 years. Exacerbations occur 2-3 times a
E. Arnold-Chiari malformation year. The patient presents with malnutrition,
pale skin, cyanosis of the lips, drumstick
31. The burns unit received a patient, who (clubbed) fingers. Tympanic percussion
6 hours ago during a fire received flame sound in the lungs, weakened respiration,
burns. On the patient’s body there is gray- numerous various moist crackles in the lower
brown area of necrosis that covers 3/4 of pulmonary segments on the left can be
the body perimeter. Occasionally there are
small blisters with hemorrhagic contents and observed. In blood: erythrocytes - 3.2·1012 /L,
patches of shredded epidermis. What local leukocytes - 8.4·109 /L, ESR- 56 mm/hour. On
therapy is necessary in this case? X-ray: lung fields are emphysematous, the
left pulmonary root is deformed and dilated.
A. Decompression necrectomy What is the most likely diagnosis?
B. Chemical necrolysis
C. Blister puncture A. Multiple bronchiectasis of the left lung
D. Necrectomy with xenotransplantation B. Chronic left-sided pneumonia
E. Necrectomy with dermal autograft C. Chronic abscess of the left lung
D. Left-sided pulmonary cystic dysplasia
32. A woman in her early- to mid-thirties E. Suppuration of the cyst in the left lung
has lost her consciousness 3-5 minutes ago.
On examination: the skin is pale, no pulse 36. A 57-year-old woman during a regular
over the carotid arteries, no spontaneous ultrasound examination presented with a
respiration, pupils are dilated; the patient space-occupying heterogeneous lesion in the
is nonresponsive, presents with atony. The right kidney. What is the most informative
patient’s condition can be determined as: method of renal tumor diagnostics?

A. Clinical death A. Spiral computed tomography


B. Natural death B. Excretory urography
C. Syncope C. Retrograde pyelography
D. Brain death D. Radioisotope renography
E. Comatose state E. Three glass urine test

33. A boy had a foreign body removed 37. A 40-year-old victim of a traffic accident
from under his nail plate. 3 days later he sustained the following injuries: closed
developed a sharp throbbing pain at the diaphyseal femur fracture, brain concussion,
end of his distal phalanx, which intensifies multiple rib fractures, hemopneumothorax,
when the phalanx is pressed, hyperemia of degloving shin injuries. What injuries require
the nail fold, elevated body temperature up the most urgent attention?
to 38.5o C , and nail plate discoloration. Make
the diagnosis: A. Multiple rib fractures, hemopneumothorax
B. Closed diaphyseal femur fracture
A. Subungual panaritium C. Brain concussion
B. Erysipelas D. Degloving shin injuries
C. Paronychia E. All injuries are equivalent
D. Erysipeloid
E. Abscess 38. At the railroad crossing a passenger
train collided with a bus. In this collision 26
34. A 32-year-old woman complains of body bus passenges died, another 18 passengers
weight loss despite her increased appetite, received mechanical injuries of varying
nervousness, and tremor of the extremities. severity. Where will be professional medical
Objectively: the skin is moist; the thyroid aid provided for the victims of this accident?
gland is diffusely enlarged, painless, soft, and Who will provide this aid?
mobile. Blood test: increased level of T3,
T4, and thyroid-stimulating hormone (THS). A. In medico-prophylactic institutions;
What is the most likely diagnosis? general physicians and surgeons
B. At the site of the accident; first-response
A. Diffuse toxic goiter emergency teams
B. Thyroid carcinoma C. At the site of the accident; specialized
C. Autoimmune (Hashimoto’s) thyroiditis second-response emergency teams
D. Thyroid adenoma D. In medico-prophylactic institutions;
E. Diffuse nontoxic goiter specialized second-response emergency teams
E. In medical institutions; all listed types of
35. A 19-year-old young man complains healthcare workers
of cough with expectoration of purulent
Хiрургiчний профiль 18

39. A 45-year-old man underwent a cardiac ambulance into the emergency hospital. He
surgery one week ago. His general state has complains of sudden pain in the lumbar area,
been deteriorating since then: dyspnea at frequent painful urination, and vomiting.
rest, retrosternal pain that irradiates to the Examination detects pain in the lumbar
neck, marked weakness. Objectively his body area, costovertebral angle tenderness, pain
temperature is hectic. His cardiac borders on palpation of kidneys and along the
are expanded, apical beat is weakened. ureter on the right. Urine test: proteins,
Auscultation detects pericardial friction rub. fresh erythrocytes, leukocytes. Make the
What is the most likely diagnosis? provisional diagnosis:
A. Acute pericarditis A. Urolithiasis, renal colic
B. Acute cardiac aneurysm B. Acute pyelonephritis
C. Myocardial infarction C. Acute glomerulonephritis
D. Acute myogenic dilatation of the heart D. Acute renal failure
E. Pulmonary embolism E. Polycystic kidney disease
40. A 45-year-old man was brought by an
Педiатричний профiль 19

1. A newborn girl has Apgar score of 7-8 A. Doppler echocardiography


points at the 1-5 minutes after birth. During B. Electrocardiography
the labor there was a brief difficulty with C. Chest X-ray
extraction of the shoulder girdle. After birth D. Rheography of the pulmonary artery
the baby presents with disturbed function of E. Ultrasound of the liver
the proximal segment and forced position
of the right arm. The shoulder is rotated 5. Mother of a 5-year-old child noticed on the
inwards, the elbow is extended, the forearm the head of her child a round ”bald” spot 3
is pronated, and the whole upper limb cm in diameter. All the hairs in the focus are
resembles an arm of a doll. What is the most broken off at the length of 5-6 mm. The day
likely clinical diagnosis in this case? before the child was petting a stray cat. Make
the diagnosis:
A. Erb-Duchenne palsy
B. Thoracic spine trauma A. Microsporia
C. Osteomyelitis of the right arm B. Superficial trichophytosis
D. Intracranial hemorrhage C. Deep trichophytosis
E. Soft tissue injury of the right arm D. Psoriasis
E. Alopecia areata
2. Disease onset was acute. A child
developed general weakness, pain in the 6. A 2-year-old child with persisting cough
joints, and elevated temperature. Later these and subfebrile body temperature after a case
signs became accompanied by itching skin of URTI developed dyspnea, cyanosis of
rash manifested as erythematous spots 2- the nasolabial triangle, percussion dullness
5 mm in size. The rash gradually turned and weakened respiration in the lower lobe
hemorrhagic. Large joints are painful and of the right lung, and a slight mediastinal
swollen; pain attacks periodically occur in displacement to the left. What pulmonary
the paraumbilical area; there are signs of pathology is likely to cause this clinical
intestinal hemorrhage. What is the most presentation?
likely diagnosis? A. Pleurisy
A. Hemorrhagic vasculitis (Henoch-Schonlein B. Emphysema
C. Pneumonia
purpura) D. Atelectasis
B. Scarlet fever E. Bronchitis
C. Hemorrhagic meningoencephalitis
D. Streptococcal impetigo 7. During examination a 4-month-old
E. Rheumatism child with meningococcemia presents with
acrocyanosis, cold extremities, tachypnea,
3. A 13-year-old girl for the last two weeks and thready pulse, blood pressure of 30/0
has been complaining of dyspnea and mm Hg, anuria, and sopor. What clinical
shin and foot edemas that appear after a syndrome is it?
physical exertion. In the morning the edemas
significantly decrease. Clinical examination A. Toxic shock syndrome
revealed enlarged liver and coarse systolic B. Neurotoxicosis
murmur over the heart area. Blood test and C. Exicosis
urinalysis are without changes. What is the D. Encephalic syndrome
most likely cause of edemas in this child? E. Acute renal failure
A. Heart failure 8. At night a 2-year-old child with
B. Nephrotic syndrome upper respiratory tract infection suddenly
C. Acute pyelonephritis developed dyspnea with labored inspiration.
D. Angioneurotic edema Objectively the skin is pale, perioral cyanosis
E. Hepatic cirrhosis and slight acrocyanosis are observed.
Breathing is loud, respiration rate is 32/min.
4. A 7-year-old boy has severe pulmonary Jugular, supra- and infraclavicular fossae
mucoviscidosis (cystic fibrosis). He retract during breathing. Respiration is
complains of dyspnea and blood coarse on auscultation. Heart sounds are
expectoration. Objectively he presents with clear and sonorous, heart rate is 120/min.
lagging physical development, acrocyanosis, What condition was complicated by the
hepatomegaly, drumstick fingers, and nail development of the upper respiratory tract
plates resembling a ”clock face”. Provisional infection?
diagnosis of chronic pulmonary heart disease
is made. What examination would be the A. Stenosing laryngotracheitis
most informative for diagnosis confirmation? B. Airway foreign body
C. Obstructive bronchitis
D. Bronchiolitis
E. Bronchial asthma
Педiатричний профiль 20

9. A 1-year-old child with a case of URTI headache. Clinical urinalysis: proteins -


suddenly developed noisy respirations with 7.1 g/L, leukocytes - 1-2 in the vision
difficult inspiration, intercostal retractions, field, erythrocytes - 3-4 in the vision field.
and barking cough on the 2nd night after During the course of treatment the edemas
the disease onset. What is the most likely gradually dissipated, headache abated,
diagnosis? diuresis normalized. Daily urine proteins -
3 g/L. Biochemical blood test: total protein -
A. Stenosing laryngotracheobronchitis 43.2 g/L, urea - 5.2 mmol/L, cholesterol - 9.2
B. Acute pulmonary inflammation mmol/L. What glomerulonephritis syndrome
C. Bronchial asthma is the most likely to be present in the patient?
D. Acute bronchitis
E. Acute bronchiolitis A. Nephrotic
B. Nephritic
10. A 10-year-old boy with symptoms of C. Isolated urinary
arthritis and myocarditis was brought to a D. Hematuric
hospital. Based on clinical examination the E. Mixed
provisional diagnosis of juvenile rheumatoid
arthritis was made. What symptom is the 14. A 3-month-old child with signs of rickets
most contributive for the diagnostics of this presents with positive Chvostek, Trousseau,
disease? and Maslov signs. One day ago the parents
witnessed a cyanotic attack in their child -
A. Reduced mobility of the joints in the the child broke into a cold sweat, the eyes
morning bulged, and respiratory arrest occurred. One
B. Regional hyperemia of the joints minute later the child drew in a loud breath
C. Affection of the large joints and the child’s condition normalized again.
D. Enlarged heart What is the cause of the described signs of
E. Increased heart rate the disease?
11. A 7-year-old girl has been twice treated A. Decrease of blood calcium levels
with antibacterial agents for urinary tract B. Increase of blood calcium levels
infection. US shows no severe renal defects. C. Decrease of blood phosphorus levels
The child presents with recurrence of D. Increase of blood phosphorus levels
leukocyturia and bacteriuria, elevated body E. Metabolic acidosis
temperature up to 38.5o C , and pain in her left
lumbar area. What examination should be 15. A newborn with gestational age of
conducted first to clarify the cause of urinary 31 weeks presents with hypotonia and
infection recurrence? depressed consciousness. Hematocrit is
35%, general cerebrospinal fluid analysis
A. Micturating cystourethrography shows increased content of erythrocytes
B. Excretory urography and protein, and low glucose. These data
C. Retrograde pyelography correspond with the clinical presentation of:
D. Immunogram
E. Radioisotope renography A. Intracranial hemorrhage
B. Meningitis
12. A child is 1 year old. After solid food C. Sepsis
was introduced into the diet, within the D. Anemia
last several months the child developed E. Intrauterine infection
loss of appetite, diarrhea with large amount
of feces, and occasional vomiting. Body 16. A newborn has Apgar score of 9. When
temperature remains normal. Body weight should this infant be put to the breast?
is 7 kg. The child is very pale, has leg edemas
and extremely distended abdomen. Feces A. In the delivery room
analysis detects high levels of fatty acids and B. After 12 hours
soaps. Diagnosis of celiac disease was made C. After 2 hours
and gluten-free diet was prescribed. What D. On the 2nd day
should be excluded from the diet in this case? E. On the 3rd day
A. Cereals - wheat, oats 17. A 3-week-old infant developed large,
B. Milk and dairy products flaccid vesicles with purulent contents on
C. Fruits the skin of chest and abdomen. The vesicles
D. Animal protein rupture quickly. Make the provisional
E. Easily digestible carbohydrates diagnosis:
13. A 7-year-old boy has been an inpatient
for 1.5 months. He had been brought to
the hospital with complaints of edemas
all over his body, low urine output, and
Педiатричний профiль 21

A. Pemphigus neonatorum A. Tetrad of Fallot


B. Vesiculopustulosis B. Coarctation of the aorta
C. Toxic erythema C. Transposition of the great vessels
D. Pemphigus syphiliticus D. Ventricular septal defect
E. Pseudofurunculosis E. Acquired valvular disease
18. 10 hours after birth a child developed 22. A 15-year-old girl complains of dizziness
jaundice, hypotonia, hyporeflexia, and and sensation of lack of air that she develops
moderate hepatosplenomegaly. Feces and in emotionally straining situations. Relief
urine are of normal color. Umbilical cord occurs after she takes corvalol. Objectively:
blood bilirubin is 51 mcmol/L due to hyperhidrosis and marble-like pattern of
unconjugated bilirubin levels. In venous the skin of her palms and feet. Clinical
blood: erythrocytes - 3.5 · 1012 /L, Нb- 140 g/L, and instrumental examination revealed no
reticulocytes - 1.5%, bilirubin - 111 mcmol/L, organic changes in the central nervous,
conjugated - 11 mcmol/L, ALT- 40 U/L, AST- cardiovascular, and respiratory systems.
30 U/L. Mother’s blood group is А(II) Rh(- What provisional diagnosis can be made?
), child’s blood group is А(II) Rh(+). What A. Somatoform autonomic dysfunction
laboratory test can confirm the diagnosis? B. Obstructive bronchitis
A. Coombs test C. Bronchial asthma
B. Viral hepatitis markers analysis D. Stenosing laryngotracheitis
C. Measurement of erythrocyte osmotic E. Acute epiglottitis
resistance 23. A 1.5-month-old child on breasfeeding
D. Erythrocytometry presents from birth with daily vomiting,
E. Measurement of glucose 6-phosphate irregular liquid foamy feces, and meteorism,
dehydrogenase levels in erythrocytes which are resistant to antibacterial and
19. A 6-month-old child on breastfeeding probiotic therapy; no increase of body mass
is hospitalized in the inpatient department. is observed. The child’s condition improved,
After the child recovers, the doctor when breastmilk was substituted with ”NAN
recommends the mother to start introducing low lactose” formula. What pathology is it?
solid food to the child’s diet. What products A. Lactase deficiency
should be introduced to the child’s diet first?
B. Intestinal lambliasis (Giardiasis)
A. Vegetable puree C. Infectious enteritis
B. Fermented dairy products D. Drug-induced enteritis
C. Grated apple E. Functional dyspepsia
D. Semolina porridge
E. Buckwheat porridge 24. A 13-year-old girl for a month has been
complaining of fatigability, dull pain in her
20. The 5-year-old child has been ill for 2 right subcostal area, abdominal distension,
weeks. Cough attacks developed first and and constipations. Abdominal palpation
were then followed by reprises. During reveals positive Kehr, Murphy, and Ortner
coughing the child’s face turns red and signs, while Desjardins and Mayo-Robson
cervical veins bulge. The cough attacks points are painless. Total bilirubin is 14.7
induce vomiting. X-ray shows intensified mcmol/L, predominantly indirect, ALT- 20
bronchial pattern. Blood test: leukocytes - U/L, AST- 40 U/L, amylase - 6.3 mmol/L.
16 · 109 /L , lymphocytes - 72%, erythrocyte Echocholecystography shows practically no
sedimentation rate - 4 mm/hour. What is the contraction of the gallbladder. Make the
most likely diagnosis? provisional diagnosis:

A. Pertussis A. Hypokinetic biliary dyskinesia


B. Obstructive bronchitis B. Hyperkinetic biliary dyskinesia
C. Pneumonia C. Chronic pancreatitis
D. Adenovirus infection D. Acute pancreatitis
E. Foreign body E. Chronic hepatitis

21. A 3-year-old child presents with 25. A 22-day-old infant developed


dyspnea that abates in the sitting position, subcutaneous red nodes from 1.0 to 1.5 cm in
occasional loss of consciousness and seizures, size on the scalp; later the nodes suppurated.
delayed physical development, cyanosis, Temperature increased up to 37.7o C ,
drumstick fingers. Echocardioscopy detects intoxication symptoms appeared, regional
aortic dextraposition, ventricular septal lymph nodes enlarged. Complete blood
defect, pulmonary artery stenosis, and right count: anemia, leukocytosis, neutrocytosis,
ventricular hypertrophy. What is the most increased ESR. What diagnosis can be made?
likely diagnosis?
Педiатричний профiль 22

A. Pseudofurunculosis Objectively her skin is pale and cold to touch,


B. Pemphigus her pupils are dilated. Blood pressure - 90/50
C. Vesiculopustulosis mm Hg. Heart rate - 58/min. What pathology
D. Scalp phlegmon occurred in this case?
E. -
A. Syncope
26. A 10-year-old boy was brought into B. Sympathicotonic collapse
the hospital with complaints of expiratory C. Paralytic collapse
dyspnea, respirations are 30/min. He explains D. Sunstroke
his state by a change in the weather E. -
conditions. For the last 4 years the boy has
been registered for regular check-ups due 29. A 13-year-old girl has 30% of excessive
to his diagnosis of third degree persistent body mass, she started to gain weight at
bronchial asthma. To provide emergency aid the age of 3. She has a family history of
for this child, first he needs to be given: obesity. Her height and sexual development
are normal for her age. The appetite is
A. Salbutamol or short-acting β 2-agonists excessive. She complains of periodical
B. Dexamethasone headaches. Blood pressure - 120/80 mm Hg.
C. Adrenaline Subcutaneous fat is evenly distributed, she
D. Euphylline (Aminophylline) has no stretch marks. There is juvenile acne
E. Claritin (Loratadine) on her face. What type of obesity is it?
27. A 3-year-old child has been brought to A. Alimentary constitutive obesity
a hospital with complaints of pain in the B. Hypothalamic obesity
legs, fever, and loss of appetite. Objectively: C. Adrenal obesity
pale skin and mucosa, hemorrhagic rash. D. Hypothalamic syndrome of puberty
Lymph nodes are enlarged, painless, dense E. Hypothyroid obesity
and elastic, not matted together. Bones,
joints, and abdomen are painful. The liver 30. An 8-year-old girl complains of frequent
and spleen are enlarged. Hemogram: Hb- 88 painful urination in small amounts and
g/L, color index - 1.3, platelets - 80 · 109 /L, urinary incontinence. The signs have
leukocytes - 25.8 · 109 /L, lymphoblasts - 70%, been present for 2 days already. She
ESR- 52 mm/hour. Make the provisional explains her disease by overexposure to
diagnosis: cold. Costovertebral angle tenderness is
absent. Complete blood count is without
A. Acute leukemia pathologies. Urine test: leukocytes - 20-30
B. Thrombocytopenic purpura in the vision field, erythrocytes - 40-50 in the
C. Acute rheumatic fever vision field, unchanged, bacteriuria. What is
D. Infectious mononucleosis the most likely diagnosis?
E. Hemorrhagic vasculitis (Henoch-Schonlein
purpura) A. Cystitis
B. Vulvitis
28. During an outdoors school event in C. Pyelonephritis
hot weather, a 10-year-old girl lost her D. Glomerulonephritis
consciousness. Body temperature - 36.7o C . E. Urolithiasis
Акушерство i гiнекологiя 23

1. A pregnant woman is 28 years old. irritability, and headaches. Her skin is


Anamnesis: accelerated labor complicated of normal color. Blood pressure - 150/90
by the II degree cervical rupture. The mm Hg, pulse - 90/min., rhythmic. The
following two pregnancies resulted in abdomen is soft and painless. Bimanual
spontaneous abortions at the terms of 12 and examination shows no uterine enlargement,
14 weeks. On mirror examination: the uterine the appendages cannot be detected. The
cervix is scarred from previous ruptures at vaginal fornices are free. What is the most
9 and 3 hours, the cervical canal is gaping. likely diagnosis?
On vaginal examination: the cervix is 2 cm
long, the external orifice is open 1 cm wide, A. Climacteric syndrome
the internal orifice is half-open; the uterus is B. Premenstrual syndrome
enlarged to the 12th week of pregnancy, soft, C. Adrenogenital syndrome
mobile, painless, the appendages are without D. Stein-Leventhal syndrome (polycystic
changes. What diagnosis can be made? ovary syndrome)
E. Uterine myoma
A. Isthmico-cervical insufficiency, habitual
noncarrying of pregnancy 5. A 30-year-old multigravida has been
B. Threatened spontaneous abortion in labour for 18 hours. 2 hours ago the
C. Incipient abortion, habitual noncarrying of pushing stage began. Fetal heart rate is clear,
pregnancy rhythmic, 136/min. Vaginal examination
D. Cervical hysteromyoma, habitual reveals complete cervical dilatation, the fetal
noncarrying of pregnancy head in the pelvic outlet plane. Sagittal
E. Cervical pregnancy, 12 weeks suture is in line with obstetric conjugate,
the occipital fontanel is near the pubis. The
2. On the day 4 after the cesarean section patient has been diagnosed with primary
a woman developed fever with body uterine inertia. What is the further tactics
temperature up to 39o C and abdominal of labor management?
pain. Pulse - 104/min. She vomited twice.
The patient is sluggish, her tongue is dry A. Outlet forceps
and has gray coating. The abdomen is B. Labour stimulation
distended. Signs of peritoneal irritation are C. Cesarean section
positive in all segments. Peristalsis cannot D. Skin-head Ivanov’s forceps
be auscultated. No passage of gas occurs. E. Vacuum extraction of the fetus
Uterine fundus is located at the level of the
navel. The uterus is painful on palpation. The 6. A woman is 40 weeks pregnant.
discharge is moderate and contains blood The fetus is in the longitudinal lie and
and pus. What is the most likely diagnosis? cephalic presentation. Pelvic size: 26-29-31-
20. Expected weight of the fetus is 4800 gram.
A. Diffuse peritonitis The labor contractions has been lasting for
B. Metroendometritis 12 hours, within the last 2 hours they were
C. Progressive thrombophlebitis extremely painful, the parturient woman
D. Pelvic peritonitis is anxious. The waters broke 4 hours ago.
E. Parametritis On external examination the contraction
ring is located 2 finger widths above the
3. A 58-year-old woman came to the navel, Henkel-Vasten sign is positive. Fetal
gynecological clinic. She complains of heart rate is 160/min., muffled. On internal
bloody discharge from her genital tracts. examination the uterine cervix is fully open,
Menopause is 8 years. Gynecological the head is engaged and pressed to the
examination: the uterus is slightly enlarged, entrance into the lesser pelvis. What is the
dense to touch, with limited mobility; the most likely diagnosis?
uterine appendages cannot be detected;
parametrium is free. Fractional curettage of A. Threatened uterine rupture
the uterine cavity yields a significant amount B. Complete uterine rupture
of medullary substance in the scrape. What is C. Hyperactive uterine contractions
the most likely diagnosis? D. Abruption of the normally positioned
placenta
A. Uterine corpus cancer E. Anatomically contracted pelvis
B. Adenomyosis
C. Chorioepithelioma 7. A 23-year-old woman came to the
D. Uterine cervix cancer gynecological clinic. She complains of pain,
E. Hormone-producing ovarian tumor itching, and burning in her vulva, general
weakness, indisposition, elevated body
4. A 48-year-old woman complains of temperature up to 37.2o C , and headache. On
disturbed menstrual cycle: her periods last examination in the vulva there are multiple
for 7-9 days and are excessively profuse vesicles up to 2-3 mm in diameter with
throughout the last half-year. She notes clear contents against the background of
occasional hot flashes in her head, insomnia, hyperemia and mucosal edema. Make the
Акушерство i гiнекологiя 24

provisional diagnosis: discharge is bloody and profuse. What is the


most likely diagnosis?
A. Genital herpes infection
B. Primary syphilis A. 12-week pregnancy, spontaneous abortion
C. Papillomavirus infection in progress
D. Vulvar cancer B. 12-week pregnancy, threatened
E. Cytomegalovirus infection spontaneous abortion
C. Disturbed menstrual cycle,
8. A woman with the pregnancy term of 8 hyperpolymenorrhea
weeks complains of elevated temperature up D. Disturbed menstrual cycle, amenorrhea
to 37.6o C , skin rash that can be characterized E. Full-term pregnancy, term labor
as macular exanthema, enlargement of
posterior cervical and occipital lymph 12. A 30-year-old woman came to the
nodes, small amount of bloody discharge gynecological department. She complains
from the genital tracts. She was examined of sharp pain in her lower abdomen and
by the infectious diseases specialist and temperature of 38.8o C . She has a history of
diagnosed with rubella. What tactics should extramarital sexual activity and 2 artificial
the obstetrician-gynecologist choose? abortions. On gynecological examination
the uterus is unchanged. The appendages
A. Abortion are bilaterally enlarged and painful. Profuse
B. Prescription of antibacterial therapy purulent discharge is being produced from
C. Prescription of antiviral therapy the vagina. What examination needs to be
D. Treatment of incipient abortion conducted to clarify the diagnosis?
E. Prescription of hemostatic therapy
A. Bacteriological and bacterioscopic analysis
9. A 16-year-old girl has primary B. Hysteroscopy
amenorrhea, no pubic hair growth, normally C. Curettage of the uterine cavity
developed mammary glands; her genotype is D. Colposcopy
46 ХY; uterus and vagina are absent. What is E. Laparoscopy
your diagnosis?
13. It is the 3rd day after the first normal
A. Testicular feminization syndrome term labor; the infant is rooming-in with the
B. Mayer-Rokitansky-Kuster-Hauser mother and is on breastfeeding. Objectively:
syndrome the mother’s general condition is satisfactory.
C. Cushing syndrome Temperature is 36.4o C , heart rate is 80/min.,
D. Sheehan syndrome BP is 120/80 mm Hg. Mammary glands
E. Cushing disease are soft and painless; lactation is moderate,
unrestricted milk flow. The uterus is dense,
10. A 46-year-old woman came to the the uterine fundus is located 3 finger widths
maternity clinic with complaints of moderate below the navel. Lochia are sanguino-serous,
blood discharge from the vagina, which moderate in volume. Assess the dynamics of
developed after the menstruation delay of 1.5 uterine involution:
months. On vaginal examination: the cervix
is clean; the uterus is not enlarged, mobile, A. Physiological involution
painless; appendages without changes. Make B. Subinvolution
the diagnosis: C. Lochiometra
D. Pathologic involution
A. Dysfunctional uterine bleeding E. Hematometra
B. Adenomyosis
C. Ectopic pregnancy 14. A 28-year-old woman complaining of
D. Submucous uterine myoma irregular menstruations and infertility came
E. Cancer of the uterine body to the gynecological clinic. Menstruations
occur since the age of 15, irregular, with
11. A 25-year-old woman was brought into delays up to 2 months. On examination she
the gynecological department with profuse presents with marked hirsutism and excessive
bloody discharge from her genital tracts. body weight. On vaginal examination the
She is 12 weeks pregnant, the pregnancy uterus is reduced in size and painless.
is planned. Within the last 3 days she was The ovaries on the both sides are dense
experiencing pains in her lower abdomen and enlarged. Ultasound shows microcystic
that eventually started resembling cramps, changes in the ovaries, the ovaries are
she developed bleeding. Her skin is pale, 5х4 cm and 4.5х4 cm in size with dense
pulse - 88/min., blood pressure - 100/60 mm ovarian capsule. Basal body temperature
Hg, body temperature - 36.8o C . Vaginal is monophasic. What is the most likely
examination: the uterus size corresponds diagnosis?
with 11 weeks of pregnancy, the cervical
canal allows inserting 1 finger and contains
fragments of the fertilized ovum, the
Акушерство i гiнекологiя 25

A. Polycystic ovary syndrome complication occurred in the patient?


B. Krukenberg tumor
C. Endometrioid cysts A. Posthemorrhagic anemia
D. Bilateral adnexitis B. Somatoform autonomic dysfunction of
E. Ovarian carcinoma hypotonic type
C. Migraine
15. An 18-year-old girl was brought into the D. Gastritis
gynecology deparment with complaints of E. Dysmenorrhea
elevated body temperature up to 37.8o C ,
sharp pain in her lower abdomen, more 19. A 22-year-old postparturient woman on
intense on the right, and difficult defecation. the 12th day after the normal childbirth
Vaginal examination detected a painful informs of elevated body temperature up to
dense elastic formation 5x6 cm in the area 39o C for the last 3 days and pain in her right
of her right ovary. Pregnancy test is negative. mammary gland. The right mammary gland
What is the most likely diagnosis? is enlarged, hot to touch, tense, hyperemic,
and painful. Palpation reveals there a dense
A. Torsion of ovarian tumor pedicle infiltration 8x8 cm with a fluctuation in its
B. Ectopic pregnancy center. What is the most likely diagnosis?
C. Appendicitis
D. Ovarian cyst rupture A. Postpartum period, day 12. Right-sided
E. Ovarian apoplexy infiltrative-purulent mastitis
B. Postpartum period, day 12. Right-sided
16. A 26-year-old woman presents with serous mastitis
amenorrhea. 10 months ago she gave birth C. Postpartum period, day 12. Right-sided
for a second time. In her early postpartum gangrenous mastitis
period she developed a massive hypotonic D. Postpartum period, day 12. Right-sided
hemorrhage. No breasfeeding. Lately she phlegmonous mastitis
has been presenting with loss of weight, E. Postpartum period, day 12. Right-sided
loss of hair, and indisposition. Gynecological lactostasis
examination revealed atrophy of the external
genitals, the uterus is abnormally small, no 20. A 35-year-old pregnant woman with
uterine appendages can be detected. What is degree 1 essential hypertension, developed
the most likely diagnosis? edemas and headache at the 33 week of her
pregnancy. Objectively her general condition
A. Sheehan syndrome (postpartum pituitary is satisfactory, blood pressure - 160/100 mm
gland necrosis) Hg, normal uterine tone. Fetal heart rate
B. Physiological amenorrhea is 140/min., rhythmic. She was diagnosed
C. Suspected progressing ectopic pregnancy with daily proteinuria - 4 g/L, daily diuresis
D. Stein-Leventhal syndrome (polycystic - 1100 mL. Creatinine - 80 mcmol/L, urea
ovary syndrome) - 7 mmol/L, platelets - 100 · 109 /L. What
E. Galactorrhea-amenorrhea syndrome complication of pregnancy occurred?
17. A 45-year-old woman came to the A. Moderate preeclampsia
maternity clinic with complaints of periodical B. Severe preeclampsia
pains in her mammary glands that start C. Mild preeclampsia
1 day before menstruation and stop after D. Hypertensive crisis
the menstruation begins. Palpation of the E. Renal failure
mammary glands detects diffuse nodes
predominantly in the upper outer quadrants. 21. A 24-year-old pregnant woman on
What is the most likely diagnosis? her 37th week of pregnancy has been
brought to the maternity obstetric service
A. Fibrocystic mastopathy with complaints of weak fetal movements.
B. Breast cancer Fetal heartbeats are 95/min. On vaginal
C. Mastitis examination the uterine cervix is tilted
D. Hyperprolactinemia backwards, 2 cm long, external orifice allows
E. Breast cyst inserting a fingertip. Biophysical profile of
the fetus equals 4 points. What tactics of
18. A 14-year-old girl came to the general pregnancy management should be chosen?
practitioner with complaints of weakness,
loss of appetite, headache, rapid fatigability.
Her last menstruation was profuse and lasted
for 14 days after the previous delay of 2
months. Objectively: the skin is pale, heart
rate is 90/min., BP is 110/70 mm Hg, Hb
is 88 g/L. Rectal examination: the uterus
and its appendages are without changes,
no discharge from the genital tracts. What
Акушерство i гiнекологiя 26

A. Urgent delivery via a cesarean section 24. A 27-year-old woman complains of


B. Treatment of placental dysfunction and foul-smelling discharge from her genital
repeated analysis of the fetal biophysical tracts, pain in her lower abdomen, and
profile on the next day elevated temperature. The complaints arose
C. Doppler measurement of blood velocity in 2 days ago. She has a history of surgical
the umbilical artery abortion at the term of 8 weeks one week
D. Urgent preparation of the uterine cervix ago. Mirror examination: the uterine cervix
for delivery is clear, external orifice produces foul-
E. Treatment of fetal distress; if ineffective, smelling discharge. Vaginal examination: the
then elective cesarean section on the next day uterus lies in anteflexion, is mobile, painful,
and slightly enlarged. The appendages are
22. During regular preventive gynecological without changes. Make the provisional
examination a 30-year-old woman was diagnosis:
detected to have dark blue punctulated
”perforations” on the vaginal portion of A. Postabortal endometritis
the uterine cervix. The doctor suspects B. Enterocolitis
endometriosis of the vaginal portion of the C. Appendicitis
uterine cervix. What investigation method D. Acute respiratory disease
would be most informative for diagnosis E. Salpingoophoritis
confirmation?
25. A 17-year-old girl has made an
A. Colposcopy, target biopsy of the cervix appointment with the doctor. She plans to
B. US of the lesser pelvis begin her sex life. No signs of gynecological
C. Hysteroscopy pathology were detected. In the family
D. Curettage of the uterine cavity history the patient’s grandmother had
E. Hormone testing cervical cancer. The patient was consulted
about the maintenance of her reproductive
23. A parturient woman is 30 years old, stage health. What recommendation will be the
I of the labor is ongoing. The fetus is in most helpful for prevention of invasive
the cephalic presentation. Auscultation of cervical cancer?
the fetal heart sounds detects bradycardia.
Evaluation of cardiotocogram yielded the A. Vaccination against human papillomavirus
following data: decrease of basal heart rate (HPV)
down to 90/min., variability - monotonous (2 B. Vitamins, calcium, omega-3
and less); late decelerations with amplitude C. Immunomodulators
of 50/min. Make the diagnosis and choose D. Antiviral and antibacterial drugs
the obstetrical tactics necessary in this case: E. Timely treatment of sexually transmitted
diseases
A. Fetal distress. Urgent cesarean section
delivery
B. Fetal distress. Vacuum extraction delivery
C. Normal condition of the fetus. Vaginal
birth
D. Fetal distress. Stimulation of uterine
contractions
E. Fetal distress. Forceps delivery
Гiгiєна та органiзацiя охорони здоров’я 27

1. A 38-year-old woman works in flax


processing, she dries flax. She came to the A. Endocrine disorders
hospital complaining of difficult breathing, B. Acceleration
constricting sensation in her chest, and cough C. Certain components of her diet
attacks. These signs appear on the first day of D. Sports training
her working week and gradually diminish on E. Deficient hygienic education
the following days. What respiratory disease
is likely in this case? 6. Employees work in conditions of high
dust concentration. Certain chemical (silicon
A. Byssinosis dioxide content) and physical properties of
B. Silicosis dust aerosols contribute to the development
C. Allergic rhinopharyngitis of occupational dust-induced diseases. What
D. Bronchial asthma is the main physical property of dust
E. Asthmatic bronchitis aerosols?
2. A district doctor has diagnosed one of A. Dispersion
his patients with dysentery. What accounting B. Magnetization
document reflects this type of morbidity? C. Electric charge
D. Solubility
A. Urgent report E. Ionization
B. Statistical report
C. Report on a major non-epidemic disease 7. In the factory cafeteria there was
D. Certificate of temporary disability an outbreak of food poisoning. Clinical
E. Control card of a patient registered for presentation indicates staphylococcal
regular check-ups etiology of this disease. 15 people are sick.
To confirm the diagnosis of food poisoning,
3. A 39-year-old man, a battery attendant, samples need to be sent to the laboratory.
suddenly developed weakness, loss of What samples should be obtained for
appetite, nonlocalized colicky abdominal analysis?
pains, and nausea. Objectively his skin is
gray; there is a pink-gray stripe on his gums; A. Vomit masses
the stomach is soft and sharply painful. Blood B. Blood for hemoculture
test detected erythrocytes with basophilic C. Blood (complete blood count)
stippling and anemia. The patient has a D. Urine
history of peptic ulcer disease of the stomach. E. Saliva
Constipation tends to occur every 3-4
days. What is the most likely provisional 8. In April during the medical examination
diagnosis? of various population groups, 27% of
individuals presented with low working
A. Saturnism (lead poisosning) ability and rapid fatigability. The following
B. Acute appendicitis symptoms were observed in the affected
C. Perforation of gastric ulcer individuals: swollen friable gingiva that
D. Acute cholecystitis bleeds when pressed, hyperkeratosis
E. Chronic alcoholism follicularis not accompanied by skin dryness.
These symptoms most likely result from the
4. A 9-month-old infant presents with following pathology:
delayed tooth eruption and fontanel closure,
weakness, and excessive sweating. What type A. C -hypovitaminosis
of hypovitaminosis is the most likely in this B. Parodontosis
child? C. A-hypovitaminosis
D. B1 -hypovitaminosis
A. Hypovitaminosis D E. Polyhypovitaminosis
B. Hypovitaminosis C
C. Hypovitaminosis B1 9. Increased general morbidity of the
D. Hypovitaminosis B6 local population is observed in the area
E. Hypovitaminosis A near a factory, where atmosphere is being
intensively polluted with sulfurous gas. What
5. A 10-year-old girl exhibits high level of effect does polluted air have on human body
physical development ( + 3σ ), her body in this case?
length increased by 10 cm within a year
(which is double the norm for her age group), A. Chronic nonspecific
the number of permanent teeth corresponds B. Acute specific
with the age norm (20), the development C. Acute nonspecific
of her secondary sex characteristics is D. Chronic specific
three years ahead of her age (Ма, Р, Ах, E. Selective
Menarche). Development rate ahead of her 10. During medical examination of high and
biological age can occur due to:
Гiгiєна та органiзацiя охорони здоров’я 28

middle school students, the doctors vere sample there is 1 dead trichinella detected
assessing correlation between biological and in 24 sections. This meat should be:
calendar age of the school students based
on the following criteria: height growth rate A. Sent for technical disposal
per year, ossification of the carpal bones, the B. Allowed for sale with no restrictions
number of permanent teeth. What additional C. Processed and sold through public catering
development criterion should be assessed at network
this age? D. Processed for boiled sausage production
E. Frozen until the temperature of -10o C is
A. Development of secondary sex reached in the deep layers, with subsequent
characteristics exposure to cold for 15 days
B. Body mass
C. Chest circumference 16. To assess the effectiveness of medical
D. Vital capacity of lungs technologies and determine the power and
E. Hand strength direction of their effect on the public health
indicators, the research was conducted to
11. During analysis of morbidity in the study the immunization rate of children and
city, it was determined that age structure measles incidence rate by district. What
of population is different in each district. method of statistical analysis should be
What statistical method allows to exclude applied in this case?
this factor, so that it would not skew the
morbidity data? A. Calculation of correlation coefficient
B. Calculation of morbidity index among the
A. Standardization nonvaccinated
B. Wilcoxon signed-rank test C. Calculation of matching factor
C. Correlation-regression analysis D. Calculation of standardized ratio
D. Dynamic time series analysis E. Calculation of statistical significance of the
E. Analysis of average values difference between two estimates
12. Clinical statistical investigation was 17. Having studied the relationship between
performed to determine effectiveness of the distance from villages to the local
a new pharmacological preparation for outpatient clinics and frequency of visits to
patients with ischemic heart disease. What the clinics among the rural population of
parametric test (coefficient) can be used to this area, it was determined that the rank
estimate the reliability of the results? correlation coefficient in this case equals -0.9.
How can this relationship be characterized?
A. Student’s t-distribution
B. Sign test A. Strong inverse relationship
C. Matching factor B. Strong direct relationship
D. Wilcoxon signed-rank test C. Moderate inverse relationship
E. Kolmogorov-Smirnov test D. Moderate direct relationship
E. -
13. In a rural health care area there is
an increasing cervical cancer morbidity 18. In the inpatient gynecological unit within
observed. The decision is made to conduct a year 6500 women underwent treatment.
a medical examination of the women living They spent there a total of 102000 bed-days.
in this locality. What type of medical What indicator of the gynecological unit
examination is it? work can be calculated based on these data?
A. Target A. Average length of inpatient stay
B. Preliminary B. Average bed occupancy rate per year
C. Regular C. Number of beds by hospital department
D. Complex D. Bed turnover rate
E. Screening E. Planned bed occupancy rate per year
14. In the process of hiring, a prospective 19. A middle school teacher with 4-year-
employee has undergone preventive medical long record of work was issued a medical
examination and was declared fit to work in certificate for pregnancy and childbirth leave.
this manufacturing environment. What type What amount of pay will she receive for the
of preventive medical examination was it? duration of her leave in this case?
A. Preliminary A. 100% of average salary
B. Scheduled B. 50% of average salary
C. Periodical C. 70% of average salary
D. Specific D. 60% of average salary
E. Comprehensive E. 80% of average salary
15. On laboratory investigation of a pork 20. In the air of the feed kitchen at the
Гiгiєна та органiзацiя охорони здоров’я 29

poultry factory, at the area where formula


feed is being mixed, the dust concentration A. Primary healthcare
reaches 200 mg/m3 . Air microflora is B. Emergency aid
represented predominantly by Asperqillus C. Secondary healthcare
and Mucor fungi. What effect determines D. Tertiary healthcare
pathogenic properties of the dust? E. Palliative care

A. Allergenic 23. Human body receives from the


B. Teratogenic atmosphere a number of chemicals. What
C. Mutagenic type of action results in the combined effect
D. Fibrogenic that is less than the sum of isolated effects of
E. Toxic these chemicals on the body?

21. During regular medical examination a A. Antagonism


lyceum student presents with signs of cheilitis B. Potentiation
that manifests as epithelial maceration in C. Isolated action
the area of lip seal. The lips are bright- D. Synergistic action
red, with single vertical cracks covered with E. Complex action
brown-red scabs. These clinical signs are 24. Clinical trials have proved the
most likely caused by insufficient content of ”Lipoflavon” drug to be effective for
the following in the diet: treatment of unstable angina pectoris in the
A. Riboflavin control group and experimental group of
B. Ascorbic acid patients. Neither patients nor researchers
C. Retinol knew who belonged to which group. Name
D. Thiamine this type of study:
E. Calciferol A. Double blind study
22. A 30-year-old woman made an B. Simple blind study
appointment with the family doctor for C. Triple-blind study
scheduled vaccination of her 2-year-old D. Total-blind study
child. What type of healthcare provides such E. Multicenter study
medical services?

You might also like